Sie sind auf Seite 1von 87

PRAVINN MAHAJAN CLASSES 9871255244, 8800684854

LEARNING CURVE
What is Learning curve?
Learning curve is essentially a measure of the experience gained in production of an article by an
organization. As more and more units are produced, workers involved in production become more
efficient than before. Each subsequent unit takes fewer man-hours to produce. The Learning curve exists
during a worker's startup or familiarization period on a particular job. After the limits of experimental
learning are reached, productivity tends to stabilize and no further improvement is possible.
-

learning curve measures the efficiency of labour (not machine)


It is effective only for new jobs
Once the job is familiarized, concept of learning curve is not effective
st
If learning curve is 80%, then if 1 unit is produced in 10 hours, two units will be produced in 16
hrs, i.e as output doubles average time per unit will be 80 % of original time. When 2 units are
produced average time per unit is 8hrs. If 4 units are produced, average time per unit is (80% of 8
hrs) 6.4 hrs per unit. Thus total time for producing 4 units is 25.6 hrs.If 8 units are produced,
average time per unit is (80% of 6.4 hrs) 5.12 hrs. Total time of producing 8 units is 40.96 hrs.

The nature of learning curve phenomenon can be described as a constant percentage reduction in the
average direct labour input time required per unit as the cumulative output doubles (increases)
LC model is the mathematical (graphical) portrayal of the decreasing rate of increase in the costs of
labour whilst experience is being gained in a new task

Cumulative average time

N o. O f units

L e a rn in g C u rv e
According to the graph, as production increases, average time per unit of output decreases. But this
reduction in average labour time per unit gradually diminishes. When total production is large enough the
time per unit becomes quite stable. The learning curve thus tapers off and gradually ceases to influence
the time taken to perform the task.
The learning curve ratio can be calculated with the help of the following formula:
Learning curve ratio = Average labour cost of first 2N units
Average labour cost of first N units

PRAVINN MAHAJAN CLASSES 9871255244, 8800684854

Enlist the applications of learning curve ?


Following are the areas where the effects of learning curve would be useful to decision making:
a. Setting up of standard costssince in the learning phase average time per unit is known due to
learning curve , it helps in setting standards in this phase. Once the job is familiarized standard
cost can be set without the application of learning curve model as time per unit will remain same.
b. Pricing decisions- Since learning curve permits better cost predictions, it should be employed in
cost predictions
c.

Capital budgeting- Most important aspect of capital budgeting is cash flows. Learning curve
suggests that unit costs are likely to begin high and reduce afterwards. It permits improved
estimates of production levels that can be attained and thus has implications on cash flows.

d. Training- once it is known that gradual experience on job will reduce average time per unit o
production, thereby average cost per unit, it is better to provide intensive training to workers to
save the initial increased cost and time. Training will lead to greater scope of learning and larger
reduction in inputs in successive attempts.
e. Marketing- the study of learning curve helps in sales projections and planning for advertisement,
delivery schedules to coincide with expected production schedules.
f.

Direct costs- Learning curve applies to an industry where there is high labour turnover or when
products and process are subject to frequent changes. A knowledge of learning curve helpps in
direct labour cost budget, as the labour hours or cost is reduced for repeat orders.

g. work scheduling - Learning curve increases a firms ability to predict their required labour input and
make it possible to forecast labour needs.

Enumerate limitations of Learning curve?

N11

1. All activities are not subject to learning effect. It measures only efficiency of labour and not any
other element of production (eg machine). It applies only to new labour or new operations.
2. It involves careful and tedious analysis of data ana valid data for computation of learning effect is
not easily available.
3. Learning curve is affected by factors other than adaptation of workers of a new jo.Such other
factors may be in the form of motivation of employees, improvement of facilities, arrangements
and equipment.
4. Any alterations in the existing circumstances renders learning curve obsolete.
5. According to learning curve effect average time per unit decreases as production increases
initially, but it is very unlikely that there will be a regular consistent rate of decrease. Thus cost
predictions based on conventional Learning curve are unreliable.
6. Learning curve assumes that production will continue without any interruptions. If for any reason
work is interrupted the curve will change its shape.

PRAVINN MAHAJAN CLASSES 9871255244, 8800684854

Explain distinctive features of learning curve theory in manufacturing envoirment

N10

The production quantity of a given item doubled the cost of that item decrease at a fixed rate. This
phenomenon is the basic premise on which the theory of learning curve has been formulated.
The distinctive features of a learning curve are:
1.
Better tooling methods are developed and used
2.
More productive equipments are designed and used to make the product.
3.
Design bugs are detected and corrected.
4.
Better design engineering reduces material and labour costs.
5.
Early teething problems are overcome. As production progresses management is prompted to
achieve better planning and better management.
6.
Rejections and rework tend to diminish over time.
7.
As quantity produced increases the Cost per unit decreases, since each unit entails:
(i) Lesser labour (Ii) Greater productivity of material and labour (III) Fewer delays
and lesser time losses.

PRAVINN MAHAJAN CLASSES 9871255244, 8800684854

Q1

The direct labour hours to assemble the first unit of a new equipment is Rs 400. Assuming that this type of
rd
th
assembly will experience a learning effect of 90%. Compute the average direct labour or 3 and 4 units and
th
th
th
th
also for the 5 to 8 units (For p = 90%, take b = -1520). Also calculate the average labour for 6 and 7 units.

Q2

A first batch of 25 transistors took a total of 20 direct labour hours. It s proposed to assemble another 40 units.
What will be the average labour per unit in this lot?. Assume that there is 85% learning rate.

Q3

A company has found that the average direct labour just after completion of X units was 26.4 hours. The
average at the end of first unit was 52 hours. If there is a learning curve effect of 85%, what has been the total
output to the date?

Q4

A company which has developed a new machine has observed that the time taken to manufacture the first
machine is 600 hours. Calculate the time which the company will take to manufacture the second machine if the
actual learning curve rate is (i) 80% and (ii) 90%. Explain which of the two learning rates will show faster
learning.
N08

Q5

A firm has received an order to make and supply 8 units of a standard product, which involves intricate labour
operations. The first unit was made in ten hours. It is understood that this type of operations is subject to 80%
learning effect. The workers are paid a wage rate of Rs 12 per hour.
1.
What is the total time and labour cost required to execute the above order?
2.
If repeat order of 24 units is also received from the same customer, What is the labour cost necessary
for the second order?

Q6

A company has made 6,000 units of a product. The labour to make each 1000 unit is as follows
Unit No. (000s)
1
2
3
4
5
6
Labour Content (person- hour) 385
344
325
310
301
292
1. Estimate the Learning rate for this product using 1000 units of the product as the unit of production.
2. Predict the labour requirements to make the next 2,000 units.

Q7

Illustrate the use of Learning curve for calculating the expected average unit cost of making:
(a)
4 machines
(b)
8 machines
Using the data given below:
Direct labour needed to make the first machine
= 1000 hours
Learning curve ratio
=
90%
Direct labour cost
= Rs 15 per labour hour
Direct material cost per unit
= Rs 1,50,000

Q8

XYZ Co. has observed that a 90% learning curve ratioapplies to all labour related costs each time a new
product enters production. It is anticipated that 320 units will be manufactured during 2002. Direct labour cost
for the first lot of 10 units amount to 1000 hours@ Rs 8 per hour. Variable overhead cost is assigned to product
@ Rs 2 per direct labour hour. Determine:
(i)
(ii)
(iii)

Total labour and labour related costs to manufacture 320 units of output.
Average cost of firstv(a) 40 (b) 80 (c) 100 units produced
Incremental cost of (a) units 41-80 and (b) units 101-200

PRAVINN MAHAJAN CLASSES 9871255244, 8800684854

Q9

Q10

A customer has asked your company to prepare a bid on supplying 800 units of a new product. Production will
be in batches of 100 units. You estimate that the cost for the first batch of 100 units will average Rs 100 a unit.
You also expect that a 90% learning curve will apply to the cumulative labour cost on this contract.
Required:
(a) Prepare an estimate of the labour costs of fulfilling this contract.
(b) Estimate the incremental labour cost of extending the production run to produce an additional 800 units.
(c) Estimate the incremental labour cost of extending the production run from 800 units to 900 units.
XYZ and company has given the following data:
Learning curve ratio
80%
No of units
1
2
3
4

Average hours
100
80
?
64

Total hours
100
160
?
256

Marginal hours
100
60
?
?

You are required to fill in the blanks.


b

Q11

The learning curve model is Y=ax where Y is the average time per unit for X units, a is the time for the first
unit, x is the cumulative number of units and b is the learning coefficient. Taking b = (Log 0.8 Log 2) = 0.322 for a learning rate is 80% and a = 10 hours, calculate- (a) Average time for 20 units, (b) Total time for
30 units, and (c) Time for units 31 to 40.
Given that
Log 2 =
0.3010
Antilog of 0.5811 = 3.812
Log 3 =
0.4771
Antilog of 0.5244 = 3.345
Log 4 =
0.6021
Antilog of 0.4841 = 3.049.

Q12

Boeing is an aircraft manufacturer that has introduced a new airplane model. It expects the plane to cost Rs
1500 lakhs to manufacture (not including development cost) but their products have historically exhibited in 85%
experience curve, Assuming the same experience rate will apply to this product, compute the cost of computing
the each of the first five planes, and yhe average cost per plane to produce the first 25 planes
RTP

Q13

Havoc company developed and manufactured a new machine. The manufacture of the first machine took 800
direct labour hours. The direct wage rate is Rs 20 per hour. The company experiences a learning curve effect of
80% (index is -0.3219). The first piece was used as a demonstration piece and was not intended for sale. On
the basis of demonstration , the company obtained an order for the manufacture of 20 machines. The direct
Material cost is Rs 16,000 per machine. The variable overhead rate is Rs 25 per direct labour hour. The fixed
overheads on absorbtion costing amounted to Rs 40 per direct labour hour. The selling price is to include a
profit margin of 20% on selling price. Subsequently , after the delivery of the 20 machines, the company
receives a repeat order for supply of 30 machines.
I.
II.

Calculate the Selling price per machine of the first lot of 20 machines.
What reduction in Selling price can the company allow in respect of the repeat order

RTP

PRAVINN MAHAJAN CLASSES 9871255244, 8800684854

Q14

A company has 10 direct workers, who work for 25 days a month of 8 hours per day. The estimated
downtime is 25% of the total available time. The company received an order for a new product. The first
unit of the new product requires 40 direct labour hours to manufacture the product. The company expects
80% (index is 0,322) learning curve for this type of work. The company uses standard absorbtion
costing and the cost data are as under:
Direct Material
Rs 60 per unit
Direct labour
Rs 6 per direct labour hour
Variable overheads
Rs 1 per direct labour hour
Fixed overheads
Rs 7,500 per month
Required:
(a)
Calculate the cost per unit of the first order of 30 units.
(b)
If the company receives a repeat order for 20 units, what price will be quoted to
yield a profit of 25% on selling price?
N02

Q15

An electronics firm which has developed a new type of fire alarm system has been asked to quote for a
prospective contract. The customer requires separate price quotations for each of the following possible ordersOrder
Number of fire alarm systems

First
100

Second
60

Third
40

The Firm estimates the following cost per unit for the first orderDirect Materials
Direct Labour: Department A (Highly automatic)
Department B (skilled Labour)
Variable overheads
Fixed overheads Department A
Department B

Rs 500
20 hours @ Rs 10 per hour
40 hours @ Rs 15 per hour
20% of direct labour
Rs 8 per hour
Rs 5 per hour

Determine a price per unit for each of the 3 orders, assuming the Firm uses a mark-up of 25% on total
costs and allows for an 80% Learning curve. Extract from 80% Learning curve table
M05.
X
Y

1.0
100,0

1.3
91.7

1.4
89.5

1.5
87.6

1.6
86.1

1.7
84.4

1.8
83.0

1.9
81.0

2.0
80.0

X represents the cumulative total volume produced to date expressed as a multiple of the initial order
Y is the Learning curve factor, for a given X value, expressed as a percentage of the cost of the initial order.
Q16

A company has just completed the manufacture of 40 units of a new product. The manufacturing costs areDirect materials
2,00,000
Direct labour hours: 8000 hours @ Rs 20 per hour
1,60,000
Variable overheads
80,000
Special tools (re-usable)
10,000
Fixed overheads apportioned
1,00,000
Total
5,50,000
The Companys policy is to add a profit of 12% on selling price.
The company received another order for 120 units of this product for which the company quoted, based
on its policy on absorbtion cost basis, a price of rs 15,625 per unit. The customer struck the order to Rs
11,000 per unit. The Company is short of work and so is keen to take up more orders but it is reluctant to
accept this order price because it is against the policy to accept any price before its cost. The company
experiences a Learning curve of 90%.
1. Compute the gain or Loss arising from acceptance of the order of Rs 11,000 per unit.
2. Advise whether the company should accept this order for 120 units or not.
RTP

PRAVINN MAHAJAN CLASSES 9871255244, 8800684854

Q17

The Gifts company makes momentos for offering chief guests and other dignitaries at functions.
A customer wants 4 identical pieces of hand-crafted gifts for 4 dignitaries invited to its function.
For this product, the gifts company estimates the following costs for the first unit of the product
Rs/unit
Direct variable cost (excluding labour)
2,000
Direct labour (20 hours @ Rs 50 per hour)
1,000
90% Learning curve ratio is applicable and one labourer works foe one customers order.
i.
What is the price per piece to be quoted for this customer if the targeted contribution is Rs
1,500 per unit?
ii.
I 4 different labourers made the 4 products simultaneously to ensure faster delivery to the
customer, can the price at (i) above be quoted? Why?
N09

Q18

PQ Ltd. Makes and sells a labour intensive product. Its labour force has a learning rate of 80%, applicable
only to direct labour and not to variable overhead.
The cost per unit of the first product is as follows:
Direct materials
10,000
Direct labours
8,000 (@ Rs 4 per hour)
Variable overheads
2,000
Total variable cost
20,000
PQ Ltd has received an order from X Ltd for 4 units of the product. Another customer, Y Ltd is also interested
in purchasing 4 units of the product. PQ Ltd has the capacity to fulfill both the orders. Y Ltd presently
purchases this product in the market for Rs 17,200 and is willing to pay this price per unit of PQs product. But
X Ltd lets PQ choose one of the following options:
i.

A price of Rs 16,500 per unit for the 4 units it proposes to take from PQ
Or
ii.
Supply X Ltds idle labour force to PQ for only 4 units of production, with PQ having to pay only
Re 1 per labour hour to X Ltd workers. X Ltds workers will be withdrawn after the first 4 units are
produced. In this case, PQ need not use its labour for producing X Ltds requirement. X Ltd
assures PQ that its labour force also has a learning rate of 80%. In this option, X Ltd offers to buy
the product from PQ at only Rs 14,000 per unit.
X and Y shall not know of each others offer.
If both orders came before any work started, what is the best option that PQ may choose?
Present suitable calculations in favour of your argument.
J09
Q19

M Ltd manufactures a special product purely carried out by manual labour. It has a capacity of 20,000 units. It
estimates the following cost structure:
Direct Material
Rs 30/ unit
Direct labour (1 hour / unit)
Rs 20 per unit
Variable overhead
Rs 10 per unit
Fixed overheads at maximum capacity are Rs 1,50,000
It is estimated that at the current level of efficiency, each unit requires one hour for the first 5,000 units.
Subsequently it is possible to achieve 80% learning rate. The market can absorb the first 5,000 units at Rs
100 per unit. What should be the minimum selling price acceptable for an order of 15,000 units for a
prospective client?
M08

PRAVINN MAHAJAN CLASSES 9871255244, 8800684854

Q20

A company with two production departments has set the following standard for the forthcoming year:-

Direct labour hours available per period


Standard wage rate per hour
Expected Learning curve
Standard variable overheads per hour
Standard fixed overheads per hour
Direct labour hours required for first unit in lot of 100 units

Department S
6,000
Rs 6
80%
Rs 9
Rs 12
18

Department W
4,000
Rs 5
70%
Rs 5
Rs 8
9

The direct materials are introduced in Department S. The company is able to negotiate the following
prices for purchase of direct materials during the year.
Level of output (units)

Price of direct materials per


Unit of output
100
Rs 72.00
200
Rs 64.80
800
Rs 54.00
Overtime, if required is paid at time and a half. The overhead rates as given above does not include
overtime premium.
It is policy of the company to add profit margin as under in quoting the prices:
Department
%age on total labour and overhead cost
S
25%
W
15%
Subcontracted work
5% on subcontract price
The company has received a special order. Special tooling costs of the order amounts to rs 1200. If this
order is for 200 units or less, it will be executed in the period which has a workload of 3840 direct labour
hours in Department S and 2100 direct labour hours in Department W. For the work which is done in
Department W, a subcontract, an associate company quotes price of Rs 50 per unit.
Required:
(1) If the company decided to get the work executed entirely within the company, what price, on cost
plus basis, should be quoted for the order, if it consists of 100, 200 units
(2) Assuming that the initial order placed by the customer is for 200 units. What lowest price should
be quoted for a repeat order of 600 units? Assume that this order will be executed when there are
no capacity constraints.
(3) State the output level at which the company should close down Department W to get the work
executed through subcontractors

PRAVINN MAHAJAN CLASSES 9871255244, 8800684854

Q21

SV Ltd. Which has a fairly full order book is approached by a customer with the offer of a contract for a
model that is a variant, in terms of dimensions and materials used, of one of its existing products.
Though the customer expects to pay a normal price for the model he wants SV Ltd to take account of an
80% Learning curve in its price calculation; this level has been shown to be reasonable in SV Ltds
industry for relevant work.
The prospective contract is for a total of 464 units made up of an initial order of 160 units, two subsequent
orders of 80 units each and three subsequent orders of 48 units each. SV Ltd estimates the following
costs for the initial order.
Direct materials:
P- 8 meter
At Rs 3.50 per meter
Q-12 kg
At Rs 1.00 per kg
Direct wages
Departments
Hours
Rs per hour
1
100
2.00
2
320
3.00
3
160
1.00
Variable overheads: 20% of Direct wages.
Fixed overheads Department Recovery rate per Hour (Rs)
Department
1
Rs 2 per hour
2
Rs 1 per hour
3
Rs 2 per hour
The nature of work in 3 production departments is as follows
Department 1 uses highly automatic machines. Although the operators on these machines need to be
fairly skilled, their efficiency only affects the quality of the work but can have little impact on the quantity of
his departments output which is largely machine controlled. Department 2 and 3 the skill of operators is a
major determinant of the volume of output.
You are required to calculate the cost per unit for:
a. The initial order of 160 units
b. The second, third and fourth orders, if given successively but without guarantee of further
orders and
c. The whole contract of six orders if given from the start but on the same basis of production
and delivery.
Note:- An 80% learning curve on ordinary graph paper would show the following relation ship between
the X axis (volume) and Y axis ( Cumulative average price of elements subjects to the learning curve)
X
Y%

1.0
100.00

1.1
96.0

1.2
93.3

1.3
91.7

1.4
89.5

1.5
87.6

1.6
86.1

1.7
84.4

1.8
83.0

1.9
81.5

2.0
80.0

X
Y%

2.1
78.9

2.2
77.8

2.3
76.8

2.4
76.0

2.5
74.9

2.6
74.0

2.7
73.2

2.8
72.3

2.9
71.5

3.0
70.7

3.1
70.0

PRAVINN MAHAJAN CLASSES 9871255244, 8800684854

LINEAR PROGRAMMING

Define linear programming

M95

Linear programming deals with the situations in which activities compete for limited resources, along with
restrictions and constraints of markets, to attain the given objective in a situation in which the constraints
(restrictions) and objective can be expressed as linear mathematical functions.
Thus Linear programming is a mathematical technique for optimum allocation of scarce resources, which
may be in the form of limited raw materials, labour hours, machine time, capital, market demand, to
achieve the specified objective which may be of cost minimization or profit maximization.
In order to apply Linear programming , there are certain requirements to be met, these are
a. There should be a defined objective which can be measured in quantitative terms. It is denoted
by function Z. It should be expressed as linear function of decision variables.
b. The activities should be distinctly identifiable and measurable in quantitative terms.
c. The resources should be limited, distinctively identifiable and measurable in quantitative terms
d. The objective function and constraint equations should be linear in nature.
Q

Enlist the assumptions under Linear programming


Linear programming model is based on following assumptions:
a. Proportionality- It exists in the objective function and constraint inequalities
- Proportionate change in output will lead to proportionate change in Profits
Eg if profit of one unit is Rs 4, then total profit is 4x1 , where x1 is the number of units of
product. If 10 units are produced profit is Rs 40 and for 20 units profit is Rs 80.
-

Proportionate change in output will lead to proportionate change in allocation and


consumption of resources
Eg if one unit takes 5 hours of labour, 10 units consume 50 hours and 20 units consume 100
hours.

b. Additivity- sum total of each activity conducted separately defines objective function and
constraint inequalities.
Eg
total profit in objective function is determined by the sum of profit contributed by each of
the products separately
Total resource used is sum total of resources used by each activity separately.
c.

Continuity- activities are continuous in time i.e after attaining optimum combination of the
alternative activities in attaining the given objective, such activities will be carried on in next
period.(fractional production is possible, fractional output will be carried forward as WIP and
completed in next period)

d. Certainity- The coefficients of objective function and constraints are known with certainity i.e profit
per unit and consumption of resource per unit is known and constant

10

PRAVINN MAHAJAN CLASSES 9871255244, 8800684854

e. Non negative values The optimum solution variables cannot be negative i.e the production of
any product cannot be negative in optimum solution

Enumerate the applications of Linear programming

M07, M03

Linear programming techniques has following applications


a. Industrial Applications
- Product mix
- Blending applications
- Production planning
- Trim applications
b. Administrative Applications
- Personnel assignment
c.

Product distribution
- Transportation applications

d. Marketing Applications
- Advertising Mix problems
e. Financial Applications
- Investment portfolio
f.

Operational scheduling Applications


- Flight scheduling applications

g. Agriculture applications
- Fertilizer allocation
h
Q

Diet ascertainment

Write short notes on limitation of linear programming

M98, M04, N07

1. Linear programming can ascertain the optimal solution to the problem only if constraints
inequalities and objective function can be expressed as linear mathematical functions. Practically
such linear relation does not always exist.
2. It is assumed that all constraints to production are identified and measured in quantitative terms,
whereas all constraints may not be quantified. So optimum solution ascertained by considering
quantifiable constraints may render the objective ineffective.
3. In LP coefficients of the objective function must be known and certain and should remain constant
during the given period, whereas practically in the open market any variable may change giing
uncertainity to such coefficients
4. In LPP fractional values are permissible in optimum solution. Rounding off fractional values may
not provide optimal solution.
5. Constraints are not always fixed. Some constraints can be removed by incurring additional costs.

11

PRAVINN MAHAJAN CLASSES 9871255244, 8800684854

6. Simplex method is difficult to understand and involve tedious calculations, making the planning
process expensive
7. Graphical method is effective only in case of two variables

Write short notes on Slack, Surplus and artificial variable


Slack variable
To ascertain the optimal solution by simplex method all constraint inequalities are to be converted into
equalities. To convert Less than inequalities into equalities, slack variable is added to equations. Slack
variable represents the idle or unused resource. The coefficient of such slack variable in constraint is
Positive (+1), it cannot be negative and in the objective function coefficient of slack variable is 0. It helps
in finding the solution by locating the identity matrix
Surplus variable
To ascertain the optimal solution by simplex method all constraint inequalities are to be converted into
equalities. To convert More than inequalities into equalities, surplus variable is reduced from equations.
It represents the excess amount of resources utilized. Since identity matrix cannot be located by these
variables, so these variables does not help in finding solution independently. Coefficient of surplus
variable in constraint is -1 and in objective function is 0.
Artificial variable
If the constraint is more than or equality constraint, then solution cannot be obtained as identity matrix
cannot be located. To find solution to the LPP, in case of such constraints artificial variables are
introduced in the constraints. It is a fictitious varialble with no economic meaning. Coefficient of Artificial
variable in constraint is +1 and in objective function it is M in case of maximization and +M in case of
minimization. Such variable will not appear in final solution. If the optimal table consist of Artificial
variable, then LP has no feasible solution.

12

PRAVINN MAHAJAN CLASSES 9871255244, 8800684854

Q1

A firm is engaged in producing two products, A and B. Each unit of product A requires 2 kg of raw material and
4 labour hours for processing, whereas each unit of product B requires :3 kg of raw material and 3 hours of
labour of the same type: Every week, the firm has an availability of 60 kg of raw material and 96 labour hours.
One unit of product A sold yields Rs 40 and one unit of product B sold gives
Rs 35 as profit. '
Formulate this problem as a linear programming problem to determine as to how many units of each of the
products should be produced per week so that the firm can eam the maximum profit. Assume that there is no
marketing constraint so that all that is produced can be sold.

Q2

A manufacturer can produce two products, A and B, during a given time period. Each of these products
requires. four different manufacturing operations: grinding, turning, assembly and testing. The manufacturing
requirements in hours per unit of product are given below for A and B.

Grinding
Turning
Assembly
Testing

A
1
3
6
5

B
2
1
3
4

The available capacities of these operation in hours for the given time period are: grinding 30; turning 60;
assembly 200; testing 200. The contribution to profit is Rs. 2 for each unit of A and Rs. 3 for each unit of B.
The firm can sell all that it produces at the prevailing market price. Determine the optimum amount of A and B
to produce during the given time period.
Q3

A toy manufacturer produces two types of dolls, A and B. Each doll of type B requires twice as much time
as required by a doll of type A. If all dolls were of type A, the company could make 2000 dolls per day.
The supply of plastic is sufficient to produce 1500 dolls per day and each type requires equal amount of it.
Doll B requires a fancy dress and only 600 fancy dresses are available per day. If company makes a
profit of Rs. 3 and Rs. 5 per doll, respectively, on doll A and B, how many of each should be produced per
day in order to maximize profit? Formulate this as a linear programming problem and solve by graphical
method.

Q4

A box manufacturer makes small and large boxes from a large piece of cardboard. The large boxes
require 4 sq. ft. per box, while the small boxes require 3 sq. ft. per box. The manufacturer is required to
make at least three large boxes and at least twice as many small boxes as large boxes. If 60 sq. ft. of
cardboard is in stock, and if the profits on the small and large boxes are Rs. 2 and Rs. 3 per box,
respectively, how many of each should be made in order to maximize the total profit? (Use graphic
method).

Q5

Using Graphic method, find the maximum value of


Z = 7x + 10y
Subject to
X + y 30,000
Y 12,000
X 6,000
X y

Q6

Maximize
Subject to

x,y 0

Z = 2x1 + x2 x3
x1 +x2
1
X1 2x2 x3 -2

x1, x2, x3 0

13

2nd yr 156

2nd yr 195

PRAVINN MAHAJAN CLASSES 9871255244, 8800684854

Q7

A firm produces three products A, B, and C, each of which passes through three departments:
Fabrication, Finishing and Packaging. Each unit of product A requires 3,4 and 2; a unit of product B
requires 5, 4 and 4, while each unit of product C requires 2, 4 and 5 hours respectively in the three
departments. Every day, 60. hours are available in the fabrication department, 72 hours in the finishing
department and 100 hours in packaging department.
The unit contribution of product A is Rs 5, of product B is Rs 10., and of product C is Rs 8.
Required:
(a)
Formulate the problem as an LPP and determine the number of units of each of the products, that
should be made each day to maximise the total contribution. Also determine if any capacity would
remain unutilized.
(b)
If the optimal solution obtained does not require the production of some product, explain as to
why such product would not be produced. In this context, indicate the quantity (quantities) of
other product/s that would be foregone for producing such product.
(c)
What would be the effect on the solution of each of the following:
(i) obtaining an order for 6 units of product"A, which has to be met.
(ii) an increase of 20 percent capacity in the fabrication department.

Q8

A firm uses three machines in the manufacture of three products. Each unit of product A requires 3 hours
on machine I, 2 hours on machine II and one hour on machine III. Each unit of product B requires 4 hours
on machine I, one hour on machine II and 3 hours on machine III, while each unit of product C requires 2
hours on each of the three machines. The contribution margin of the three products is Rs 30, Rs 40 and
Rs 35 per unit respectively. The machine hours available on three machines are 90, 54 and 93
respectively.
(i)
Formulate the above problem as a linear programming problem.
(ii)
Obtain optimal solution to the problem by using the simplex method. Which of the three products
shall not by produced by the firm? Why?
(iii)
Calculate the percentage of capacity utilisation in the optimal solution.
(iv)
What are the shadow prices of the machine hours?
(v)
Is the optimal solution degenerate?

Q9

Minimize
Subject to

Z = 40x1 + 24 x2

Total cost

20x1 + 50x2 4800


80x1 + 50 x2 7200
X1, x2 0
Q10

Q11

Ashok Chemicals Company manufactures two chemicals A and B which are sold to the manufacturers of
soaps and detergents. On the basis of the next month's demand, the management has decided that the
total production for chemicals A and B should be at least 350 kilograms. Moreover, a major customer's
order for 125 kilograms of product A must also be supplied. Product A requires 2 hours of processing time
per kilogram and product B requires one hour of processing time per kilogram. For the coming month,
600 hours of processing time are available.
The company wants to meet the above requirements at minimum total production cost. The production
costs are Rs 2 per kilogram for product A and Rs 3 per kilogram for product B.
Ashok Chemicals Company wants to determine its optimal product mix and the total minimum cost
relevant thereto.
(i) Formulate the above as a linear programming problem.
(ii) Solve the problem with the simplex method.
(iii) Does the problem have multiple optimal solutions? Why?
Minimize
Subject to

Z = - 4x1 + 3x2
X1 2x2 -4
2x1 + 3x2 13
X1 x2 4

x1, x2 0

14

PRAVINN MAHAJAN CLASSES 9871255244, 8800684854

Q12

Maximise
Subject to

Z = 8x1 4x2
4x1 + 5x2 20
-x1 + 3x2 - 23

Q13

Minimize
Subject to

x1 0, x2 unrestricted in sign

Z = x1 + x2 + x3
X1 3x2 + 4x3 = 5
X1 2x2
3
2x2 x3
4

Q14

Maximize
Subject to

x1, x2 0, x3 unrestricted in signs

Z = 4x1 + 5x2 3x3


X1 + x2 + x3
= 10
2x1 + 3x2 + x3 30
X1 x2
1

Q15

Q16

x1, x2, x3 0

The standard weight of a special purpose brick is 5 kg and it contains two basic ingredients Bl and B2. Bl
costs Rs. 5 per kg and B2 costs Rs. 8 per kg. Strength considerations dictate that the brick contains not
more than 4 kg of Bl and minimum of 2 kg of B2. Since the demand for the product is likely to be related to
the price of the brick, find the minimum cost of brick satisfying the above conditions.
Maximize
Subject to

Z = x1 + 3x2 2x3
-X1 2x2 2x3 = -6
-x1 x2 + x3 -2
X1, x2, x3 0

Q17

Q18

Change the following LPP suitably so that an initial solution to it mav be obtained by applying the simplex
method
Maximize
Z = 4x1 + 2x2 + 10x3
Subject to
X1 + 3x2 x3 = 11
3x1 + 2x2 + 4x3 = 54
2x1 + 5x2 + x3 = 39
x1, x2, x3 0
Maximize
Subject to

Z = 3x1 + 2x2

-x1 + 2x2
3x1 + 2x2
X1 x2
Q19

Q20

4
14
3

x1, x2 0

A company produces three products. P1, P2 and P3 from two raw materials A and 8, and labour L. One
unit of product P1 requires one unit of A, 3 units of 8 and 2 units of L. One unit of product P2 requires 2
units of A and 8 each, and 3 units of L, while one unit of P3 needs 2 units of A, 6 units of 8 and 4 units of
L. The company has a daily availability of 8 units of A, 12 units of 8 and 12 units of L. It is further known
that the unit contribution margin for the products is Rs 3, 2 and 5 respectively for P1, P2 and P3.
Formulate this problem as a linear programming problem, and then solve it to determine the optimum
product mix. Is the solution obtained by you unique? Identify an alternate optimum solution, if any.
Maximize
Subject to

Z = 8x1 + 16x2
X1 + x2
X2
3x1 + 6x2

15

200
125
900

x1, x2 0

PRAVINN MAHAJAN CLASSES 9871255244, 8800684854

Q21

Maximize
Subject to

Z = 20x1 + 30x2
2x1 + x2
4x1 x2
X1

Q22

Maximize
Subject to

Maximize
Subject to

Maximize
Subject to

8
36

16
15

Maximize
Subject to

Maximize
Subject to

Maximize
Subject to

Maximize
Subject to

X1, x2 0

18
8
30

X1, x2 0

3
4

x,y 0

Z = 5x1 + 2x2
4x1 + 2x2 16
3x1 + x2 9
3x1 x2 9

Q29

24
-3

Z = 80x + 100y
20x + 30y
60x + 40y

Q28

X1, x2, x3 0

Z = 28x1 + 30 x2
6x1 + 3x2
3x1 + x2
4x1 + 5x2

Q27

5
2
3

Z = x1 + x2
3x1 + 2x2
-x1 + 3x2

Q26

X1, x2 0

Z = 5x1 + 6x2 + x3
9x1 + 3x2 2x3
4x1 + 2x2 x3
X1 4x2 + x3

Q25

X1, x2 0

Z = 10x1 + 20x2
2x1 + 4x2
X1 + 5x2

Q24

x1, x2 0

Z = x1 + 2x2
X1 + x2
4x1 + 3x2

Q23

40
20
30

X1, x2 0

A company makes two kinds of leather belts, A and B. Belt A is a high quality bell, and B is of lower
quality. The respective profits are Rs. 4 and Rs. 3 per belt. Each belt of type A requires twice as much
time as a belt of type B, and, if all belts were of type B, the company could make 1000 per day. The
supply of leather is sufficient for only 800 belts per day (both A and B combined). Belt A requires a fancy
buckle, and only 400 per day are available. There are only 700 buckles a day available for belt B.
What should be daily production of each type of belt? Formulate the problem as a linear programming
problem.

16

PRAVINN MAHAJAN CLASSES 9871255244, 8800684854

Q30

A company produces 2 types of leather belts - Type A and B. Contribution per belt is Rs. 4 for type A and
Rs. 3 for type B. The time requirements of one belt of type A and type B are is the ratio 2 : 3. Time
available is sufficient to produce 500 belts of type A. The leather is sufficient for only 400 belts. Belt A
required a fancy buckle and only 200 fancy buckles are available.
(i) Formulate above as a linear programming problem.
nd
(ii) Solve it by simplex method and comment on this optimum solution.
2 yr 202

Q31

A company makes two kinds of leather belts. Belt A is a high quality belt, and belt B is of lower quality.
The respective profits are Rs 0.40 and Rs 0.30 per belt. Each belt of type A requires twice as much time
as belt of type B, and if all belts were of type B, the company could make 1000 per day. The supply of
leather is sufficient for only 800 belts per day (both A and B combined). Belt A requires a fancy buckle,
and only 400 per day are available. There are only 700 buckles a day available for belt B.
What should be the daily production of each type of belts? Form the linear programming problem and
hence solve it by simplex method

Q32

A retired person wants to invest upto an amount of Rs 30,000 in fixed income securities. His broker recommends
investing in two bonds : Bond A yielding 7% and Bond B yielding 10%. After some consider ion, he decides to invest
at most Rs 12,000 in Bond B and at least Rs 6,000 in Bond A. He also wants t e amount invested in Bond A to be at
least equal to the amount invested in Bond B. What should the broker recommend if the investor wants to maximise
his retum on investment? Solve graphically.

Q33

An agriculturist has a 125-acre farm. He produces radish, muttar and potato. Whatever he raises is fully
sold in the market. He gets Rs 5 for radish per kg, Rs 4 for muttar per kg and Rs 5 for potato per kg. The
average per acre yield is 1500 kg of radish, 1800 kg of muttar and 1200 kg of potato. To produce each
100 kg of radish and muttar and 80 kg of patato, a sum of Rs 12.50 has to be used for manure. Labour
required for each acre to raise the crop is 6 man-days for radish and potato each and 5 man-days for
muttar. A total of 500 man-days of labour at a rate of Rs 40 per man-day are available.
Formulate this as a Linear programming model to maximize the agriculturist's total profit.

Q34

A firm assembles and sells two different types of outboard motors, A and B, using four resources. The
production process can be described as follows:
Resources
Capacity per month
Motor unit shop resource
400 Type A units or 250 Type B units or any linear
combination of the two
Type A gear and drive
175 Type A units
shop resource
Type B gear and drive
225 Type B units
shop resource
Final assembly resource
200 Type A units or 350 Type B units or any linear combination of the two
Type A units bring in a profit of Rs. 90 each and Type B units Rs. 60 each. Formulate the above as a
linear programming problem to maximize profit and solve the same by graphic method.

Q35

A firm buys casting of P and Q type of parts and sells them as finished products after machining, boring
and polishing. The purchasing costs for casting are Rs. 3 and Rs. 4 each for parts P and Q respectively
and selling costs are Rs. 8 and Rs. 10 respectively. The per. hour capacity of machines used for
machining, boring and polishing for the two products is given below:

Machine
Boring
Polishing

P
30
30
45

Capacity per hour


or
0r
or
or

Q
50
45
30

The running cost for machining, boring and polishing are Rs. 30, Rs. 22.5 and Rs. 22.5 per hour
respectively. Formulate the linear programming problem to find out the product mix to maximize the profit.
DO NOT SOLVE.

17

PRAVINN MAHAJAN CLASSES 9871255244, 8800684854

Q36

A city hospital has the following minimal daily requirement for nurses:
Period
1
2
3
4
5
6

Clock time (24 hours day)

Minimal number of nurses required

6 am 10 am
10am - 2 pm
2pm - 6 pm
6pm - 10 pm
10pm - 2 am
2am - 6 am

2
7
15
8
20
6

Nurses report to the hospital at the beginning of each period and work for S consecutive hours\.
The hospital wants to determine the minimal number of nurses to be employed so that there will be
sufficient number of nurses available for each period. Formulate this as a Linear Programming Problem
by setting up appropriate constraints and objective function. Do not solve.
Q37

A person is interested in investing Rs 50,00,000 in a mix of investments. The investment choices and
expected rates of returns on each one of them are:
Investment
Mutual Fund A
Mutual Fund B
Money Market fund
Government Bonds
Share Y
Share X

Expected rate of return


0.12
0.09
0.08
0.085
0.16
0.18

The investor wants at least 35 per cent of his investment in government bonds. Because of the higher
perceived risk of the two shares, he has specified that the combined investment in these not to exceed
Rs. 80,000. The investor also has specified that at least 20 per cent of the investment should be in the
money market fund and that the amount of money invested in shares should not exceed the amount
invested in mutual funds. His final investment condition is that the amount invested in mutual fund A
should be no more than the amount invested in mutual fund B. The problem is to decide the amount of
money to invest in each alternative so as to obtain the highest annual. total return. Formulate the problem
as a linear programming
Q38

A company manufacturing television sets and radios has four major departments: chassis, cabinet,
assembly and final testing. Monthly capacities are as follows:

Chassis
Cabiinet
Assemble
Testing

Television capacity
1500
1,000
2,000
3,000

or
or
0r
or

Radio capacity
4,500
8,000
4,000
9,000

The contribution of television is Rs. 150 each and the contribution of radio is Rs. 25 each. Assuming that the
company can sell any quantity of either product, determine the optimal combination of output.

18

PRAVINN MAHAJAN CLASSES 9871255244, 8800684854

Q39

A pharmaceutical company has 100 kg of A, 180 kg of B and 120 kg of C available per month. They can use
these materials to make three basic pharmaceutical products, viz., 5 -10 - 5, 5 - 5 - 10 and 20 - 5 - 10, where
the numbers in each represent the percentage by weight of A, B' and C respectively in each of the products.
The cost of these ingredients arc given below:
Ingredient
A
B
C
Inert ingredients

Cost per kg (Rs.)


80
20
50
20

Selling prices of these products are Rs. 40.50, Rs. 43 and Rs. 45 per kg respectively. There is a capacity
restriction of the company for the product 5 - 10 - 5 so as they cannot produce more than 30 kg per month.
Determine how much of each of the products they should produce in order to maximize their monthly profit.
Q40

An advertising agency wishes to reach two types of audience!). Customers with annual incomes greater
than Rs. 40.000 (target audience A) and customers with annual incomes of less than Rs. 40,000 (target
audience B). The total advertising budget is Rs. 2.00,000. One programme of TV advertising costs Rs.
50,000, one programme of radio advertising costs Rs. 20,000. For contract reasons, at least 3
programmes ought to be on TV and the number of radio programmes must not exceed 5. Survey
indicates that a single TV programme reaches 4,50,000 customers in target audience A and 50,000 in
target audience B. One radio programme reaches 20,000 in target audience' A and 80,000 in target
audience B. Determine the media mix to maximize the total reach.

Q41

The daily Florist company is planning to make up floral arrangements for the upcoming festival. The
company has available the following supply of flowers at the costs shown:
Type
Red roses
Garnenias
Carnations
White roses
Yellow roses

Number available
800
456
4,000
920
422

Cost per flower(Rs.)


0.20
0.25
0.15
0.20
0.22

These flowers can be used in any of the four popular arrangements whose make up and
selling prices are as follows:
Arrangement

Requirements

Selling price

Economy

4 red roses
2 garnenias
8 carnations

Rs.6

May time

8 white roses
5 garnenias
10 carnations
4 yellow roses

Rs 8

Spring colour

9 red roses
10 carnations
9 white roses
6 yellow roses

Rs 10

Deluxe rose

12 red roses
12 white roses
12 yellow roses

Rs 12

How many units of each arrangement should be made up in order to maximize profits, assuming all
arrangements can be sold.

19

PRAVINN MAHAJAN CLASSES 9871255244, 8800684854

Q42

Hifashion Furnishers manufacture two models of dining table P and Q. The market standing of the fir~ is
such that they are able to sell at profit margins of Rs. 300 and Rs. 400 respectively of as many numbers
of P and Q as they make. The dining tables are manufactured in two stages, viz, carpentry followed by
paint shop whose capacities per month are as follows:

Carpentry:

70 of model P only
or
120 of model Q only .
or
Some intermediate mix

Paint Shop:

90 of model P only
or.
60 of model Q only
or
Some intermediate mix

Within these manufacturing capacity constraints, find how many of each model should be manufactured per
month so that the profit of the firm is maximized. FORMULATE the problem and DO NOT SOLVE.

Q43

A local travel agent is planning a charter trip to a major sea port. The eight day/seven night package
includes the fare for round trip, surface transportation, board and lodging and selected tour options. The
charter trip is restricted to 200 persons and the past experience indicates that there will not be any
problem for getting 200 clients. The problem for the travel agent is to determine the number of Deluxe,
Standard and Economy tour packages to offer for this charter. These three plans differ according to
seating and service for the flight, quality of accommodations, meal plans and tour options. The following
table summarizes the estimated prices for the three packages and the corresponding expenses for the
travel agent. The travel agent has hired an aircraft for the flat fee of Rs 2,00,000 for the entire trip.
In planning the trip, the following considerations must be taken into account:
(i)
At least 10% of the packages must be of the Deluxe type.
(ii)
At least 35% but not more than 70% must be of the Standard type.
(iii)
At least 30% must be of the Economy type.
(iv)
The maximum number of Deluxe packages available in any aircraft is restricted to 60.
(v)
The Hotel desires that at least 120 of the tourists should be on the Deluxe and Standard
packages taken together.
Tour Plan
Deluxe
Standard
Economy

Price
10,000
7,000
6,500

Hotel Costs
3,000
2,200
1,900

Meal and other expenses


4,750
2,500
2,200

The travel agent wishes to determine the number of packages to offer in each type so as to maximize the
total profit.
(a)
Formulate this as a linear programming problem.
(b)
Restate the above LPP in terms of two decision variables, taking advantage of the fact
that 200 packages will be sold.
(c)
Find the optimal solution using graphical method for the restated problem and interpret
your results.

20

PRAVINN MAHAJAN CLASSES 9871255244, 8800684854

Q44

Let us assume that you have inherited Rs 100,000 from your father-in-law that can be invested in a
combination of only two stock portfolios, with the maximum investment allowed in either portfolio set at Rs
75,000. The first portfolio has an average retum of 10%, whereas the second has 20%. In terms of risk
factors associated with these portfolios, the first has a risk rating of 4 (on a scale from 0 to 10), and the
second has 9. Since you want to maximise your retum, you will not accept an average rate of retum
below 12% or a risk factor above 6. Hence, you then face the important question. How much should you
invest in each portfolio?
Formulate this as a linear programming problem and solve it by graphic method.

Q45

A furniture manufacturer produces two types of desks: Standard and Executive. These desks
are sold to an office furniture wholesaler, and for all practical purposes, there is an unlimited market for any
mix of these desks, at least within the manufacturer's production capacity. Each desk has to go through four
basic operations: cutting of the lumber, joining of the pieces, pre-finishing, and final finish. Each unit of the
Standard desk produced takes 48 minutes of cutting time, 2 hours of joining, 40 minutes of pre-finishing, and 5
hours and 20 minutes of final finishing time. Each unit of the Executive desk required 72 minutes of cutting, 3
hours of joining, 2 hours of pre-finishing and 4 hours of final finishing time. The daily capacity for each
operation amounts to 16 hours of cutting, 30 hours of joining, 16 hours of pre-finishing and 64 hours of final
finishing time. The profit per unit produced is Rs 40 for the Standard desk and Rs 50 for the Executive desk.
Determine the product mix that maximizes total revenue.

Q46

Clarified solutions is in the process of drawing up a Capital Budget for the next three years. It has funds to
the tune of Rs. 100000 which can be allocated across the projects A, B, C, D and E. The net cash flows
associated with an investment of Re.1 in each project are provided in the following table:

From inv. A
From inv. B
From inv. C
From inv. D
From inv. E

0 i.e present

Cash Flow at Time


1year from now
2year from now

-Re. 1
Re. 0
-Re. 1
-Re. 1
Re. 0

+Re. 0.5
-Re. 1
+Rs. 1.2
Re. 0
Re. 0

+Re. 1
+Re. 0.5
Re. 0
Re. 0
-Re. 1

3year from now


Re. 0
+Re. 1
Re. 0
+Rs. 1.9
+Rs. 1.5

To ensure that the firm remains reasonably diversified, the firm will not commit an investment exceeding
Rs. 75000 in any project. The firm cannot borrow funds; therefore the cash available for investment at any
time is limited to cash on hand. The firm will earn interest.at 8% per annum by parking the uninvested
funds in money market instruments. Assume that the returns from investments can be immediately
reinvested. For example, the positive cash flow received from project C at time 1 can immediately re
reinvested in project B.
Required: Formulate an LP that wiIl Maximize cash on hand 3 years from now

21

PRAVINN MAHAJAN CLASSES 9871255244, 8800684854

Q47

A leading Firm of Chartered Accountants Is attempting to determine a best investment portfolio and is
considering six alternative Investment proposals. The following table Indicates point estimates for the
price per share, the annual growth rates in the price per share, the amount of dividend per share and a
measure of the risk associated with each investment

Particulars
Current price per share (Rs)
Projected annual growth
Projected annual dividend per share (Rs)
Projected risk in return

A
80
0.08
4.00
0.05

Shares under consideration


B
C
D
E
100 160
120
150
0.07 0.10
0.12
0.09
4.50 7.50
5.50
5.75
0.03 0.10
0.20
0.06

F
200
0.15
0.00
0.08

The total amount available for investment is Rs.25 Lakhs and the following conditions are to be satisfied.
The maximum rupee amount to be invested in alternative F is Rs.2,50,000
Not more than Rs.5,00,000 should be invested in alternatives A and B combined.
Total weighted risk should not be greater than 0.10 where
Total weighted risk
= (Amount invested in Alternative j) x (Risk of Alternative j)
Total Amount invested in all alternatives
For the sake of diversity, at least 100 shares of each stock should be purchased.
At least 10% of the total investment should be in alternatives A and B combined.
Dividends for the year should be at least Rs.10,000
Rupees Return per share is defined as the price per share one year hence less current price per share
plus dividend per share. If the objective is to maximize total rupee return, formulate (but do not solve) the
LPP determining the optimum number of shares to be purchased in each of the shares under
consideration. Assume that the time horizon for the investment is one year.
N91
Q48

Kali has 2 plants. Orders from 4 customers have been received. The number of units ordered by each
customer and the shipping cost of each plant are shown in the following table.
Customer

Units ordered

Shipping cost
Plant 1

Plant 2

A
500
Rs 15 p.u
Rs 40 p.u
B
300
Rs 20 p.u
Rs 30 p.u
C
1000
Rs 30 p.u
Rs 25 p.u
D
200
Rs 35 p.u
Rs 20 p.u
Each unit of the product must be machined and assembled. These cost, together with the capabilities at
each plant are shown below
Hours / unit
Cost (Rs / hr) Hours available
Plant No. 1

Machining
Assembling

0.10
0.20

40
30

120
260

Plant No.2

Machining

0.11
0.22

40
30

140
250

Formulate LP problem to minimize cost.

RTP

22

PRAVINN MAHAJAN CLASSES 9871255244, 8800684854


Q49

The management accountant of Atul enterprises Ltd has suggested that a Linear Programming model
might be used for selecting the best mix of 5 possible products A, B, C, D and E. The following
information is available

Particulars
Selling Price
Costs:

Materials
Direct Labour
**
Fixed overheads
Total Costs
Net Profit
**

A
48

B
42

C
38

15
18
9

14
16
8
38
4

16
16
3
25
13

42
6

Per unit of Product (Rs)


D
31
15
4
2
21
10

E
27
16
4
2
22
5

Based on 50% of direct labour cost


a. Expected Maximum unit demand per week for each product at the prices indicated
A
1,500

B
1,200

C
900

D
600

E
600

b. Cost of materials includes a special component, which is in short supply; it costs Rs 3 p.a unit.
Only 5800 units will be available to the company during the week. The number of unit of the
special component needed for a unit of each product is:
A
1

B
1

C
3

D
4

E
5

c. Labour is paid at the rate of Rs 1.50 per hpur and only 20,000 hours will be available in a week.
d. The company has ruled that expenditure on materials must not exceed a sum of Rs 30,000
e. All other resources are freely available in sufficient quantities for planned needs.
Formulate a Linear Programming model stating clearly the criterion you use

23

RTP

PRAVINN MAHAJAN CLASSES 9871255244, 8800684854

Q50

The most recent audited summarized Balance Sheet of Best Yield Financial Services is given below
Liabilities
Equity share capital
Reserves and Surplus

Rs in Lakhs
65
110

Term loan from IFCI


Public deposits
Bank Borrowings
Other current Liabilities

80
150
147
50

Assets
Fixed Assets
- Assets on lease (Cost 550 lakhs)
- Other fixed assets
Investment in wholly owned subsidiaries
Currents Assets
- Stock on hire
- Recievables
- Other current assets
- Misc expenditure not written off

602

Rs in Lakhs
375
50
20
80
30
35
12
602

The Company intends to enhance its investment in the lease portfolio by another Rs.1000 Lakhs. For this
purpose, it would like to raise a mix of debt and equity in such a way that the overall cost of raising
additional funds is minimized. The following constraints apply to the way the funds can be mobilized ,
Total debt divided by net owned funds cannot exceed 10.
Amount borrowed from financial institutions cannot exceed 25% of the Net Worth.
Maximum amount of bank borrowings cannot exceed three times the Net Owned Funds
The Company would like to keep the Public Deposit limited to 40% of the total debt.
The post tax costs of the different sources of finance are as follows
Equity 25%
Term Loans
8.5%
Public Deposits 7%
Bank borrowings
10%
.
Formulate the above as a LPP.
Q51

RTP

Welltype Manufacturing Company produces three types of typewriters: Manual typewriters, Electronic
typewriters, and Deluxe Electronic typewriters. All the three models are required to be machined first and
then assembled. The time required for the various models are as follows:
Types
Manual Typewriter
Electronic Typewriter
Deluxe Electronic Typewriter

Machine Time
15 hours
12 hours
14 hours

Assembly time
4 hours
3 hours
5 hours

The total available machine time and assembly time are 3,000 hours and 1,200 hours respectively. The
data regarding the selling price and variable costs for the three types are:
Type
Manual
Electronic
Deluxe Electronic
Selling Price (Rs.)
4,100
7,500
14,600
Labour, Material and other Variable Costs (Rs.). 2,500
4,500
9,000
The Company sells all the three types on credit basis, but will collect the amounts on the first on next
month. The Labour, Material and other Variable Expenses will have to be paid in cash.
The Company has taken a loan of Rs.40,000 from a Co-Operative Bank and will have to repay it to the
bank on April 200X. The TNC Bank from whom the Company has borrowed Rs.60,000 has expressed its
approval to renew the loan.
st

The Balance Sheet of the Company as on 31 March 200X is as follows:


Liabilities
Rs
Assets
Equity share capital
1,50,000
Land
Capital Reserve
15,000
Building
General reserve
1,10,000
Plant & Machinery
Profit and Loss A/c
25,000
Furniture & Fixtures
Long term loan
1,00,000
Vehicles
Loan from TNC bank
60,000
Inventory
Loan from TNC bank
40,000
Receivables
Cash
5,00,000

24

Rs
90,000
70,000
1,00,000
15,000
30,000
5,000
50,000
1,40,000
5,00,000

PRAVINN MAHAJAN CLASSES 9871255244, 8800684854

The Company will have to pay Rs.10,000 towards salary for top management executives and other fixed
overheads for the month. Interest on long-term loans is to be paid every month at 24% per annum. Interest on
loans from TNC and Co-operative Banks may be taken to be Rs. 1,200 for the month. Also the Company has
promised to deliver 2 Manual Typewriters and 8 Deluxe Electronic Typewriters to one of its valued customers
next month. Make sure that the level of operations in the Company is subject to the availability of cash next
month. The Company will also be able to sell all their types of typewriters in the market. The Company
desires to know as to how many units of each type writer must be manufactured in the factory next month so
as to maximise the profits of the Company. Formulate as a LPP.
M93
Q52

A Company must produce two products over a production period of three months. The Company can pay
for materials and labour from two sources: Company Funds and Borrowed Funds. The firm faces three
decisions:
(1)
(2)
(3)

How many units of Product 1 should it produce?


How many units of Product 2 should it produce?
How much money should it borrow to support the production of the products 1 and 2?

In making these decisions, the firm wishes to maximise the profit contribution on the following conditions.
Since the Company's products are enjoying a seller's market, it can sell as many units as it can
produce. The Company would therefore like to produce as many units as possible subject to
production capacity and financial constraints. The capacity constraints, to ether with cost and rice
data, are given below.
Products
Selling price
Production cost
Required hours per unit in department
(Rs per unit)
(Rs per unit)
A
B
C
1
14
10
2
11
8
Available hours per production period of 3 months

0.5
0.3
500

0.3
0.4
400

0.2
0.1
200

The available Company funds during the production period will be Rs.3 Lakhs.
A bank will give loans upto Rs.2 Lakhs per production period at an interest rate of 20% provided
the Company's Acid Test ratio is at least 1 to 1 while the loan is outstanding. Acid-test ratio is
given by
Surplus cash on hand after oroduction + Accounts Receivable
Bank borrowings + Interest accrued thereon
Also make sure that the needed funds are made available for meeting the production costs.
.
Formulate the above as a LPP.
Q53

N92

The Southern Zone of Corporation Bank is trying to determine Its overall loan policy for the region for the
coming month. Its customers can be classified into four groups: industrial, agricultural, personal loans
without solidarity and personal loans with solidarity. The total amount available for loans in the coming
month is estimated at Rs. 5 Crores. The interest rates charged and the percentage of bad debts are given
in following table
Customer type
Industrial
Agricultural
Personal loans without solidarity
Personal loans with solidarity

Interest rate charged p.a (%)


9
10
13
14

Risk level (% of bad debts)


0.2
0.5
1.0
2.0

There are a number of restrictions on loan policy, which the region must observe, due to the regulations
set by the Reserve Bank of India and .the national policy of Corporation Bank. These can be summarised
as follows:

25

PRAVINN MAHAJAN CLASSES 9871255244, 8800684854

Personal loans must not exceed 40 per cent of the value of total loans.
Personal loans with solidarity must not exceed 20 per cent of the total personal loans.
Total agricultural loans must not exceed Rs. 10 Lakhs and Industrial loans should not be less
than Rs. 2 Crores.
The average risk factor must not exceed 0.8 per cent.
(a)
(b)

Formulate a Linear Programming Model for the ,bank's problem. State clearly in your answer how
did you derive your objective function and constraints.
If you were to solve this problem by the simplex method, explain (but do not solve) how you
would deal with this situation where you have inequalities of both types, (i.e., "greater than" and
"less than").

Q54

A Mutual Fund Company has Rs.20 Lakhs available for investment in Government Bonds, Blue Chip
Stocks, Speculative Stocks and Short-Term Bank Deposits. 'The annual expected return and risk factor
are given below
Types of investments
Annual expected return (%)
Risk factor (0 to 100)
Government Bonds
14
12
Blue chip stock
19
24
Speculative stock
23
48
Short term deposits
12
6
The Company is required to keep at least Rs.2 Lakhs in short-term deposits and not to exceed an average
risk factor of 42. Speculative stocks must be at most 20 % of the total amount invested. How should the
Company invest the funds so as to maximise its total expected annual return? Formulate as a LPP

Q55

The portfolio manager of Morgan Stanley has been asked to invest Rs 10,00,000 of a large pension fund. The
investment research department has identified six mutual funds with varying investment strategies, resulting
in different potential returns and associated risks, as given below
Fund
1

Price
45
76
110
Expected return (%)
30
20
15
Risk category
high
high
high
The management has specified the following guidelines
(a)
(b)
(c)
(d)
(e)

17
12
medium

23
10
medium

22
7
low

The total amount invested in high risk funds must be between 50% and 75% of the portfolio
The total amount invested in medium risk funds must be between 20% and 30% of the portfolio
The total amount invested in low risk funds must be at least 5% of the portfolio
The amount invested in the high risk funds 1,2 and 3 should be in the ratio 1:2:3 respectively
The amount invested in the medium risk funds 4 and 5 should be 1:2

With these guidelines, what portfolio should you recommend so as to maximize the expected rate of
return? Formulate the LP problem.
Q56

Write the Dual to the following LPPs


a. Maximise
Subject to

Z = 20x1 + 15x2 + 18x3 + 10x4


4x1 3x2 + 10x3 + 4x4 60
X1 + x2 + x3
= 27
-x2 + 4x3 +7x4
35

26

x1,x2,x3 0, x4 : unrestricted in sign

PRAVINN MAHAJAN CLASSES 9871255244, 8800684854

b. Maximise
Subject to

Z = 4x1 + 3x2
3x1 x2
X1 + x2
- 4x1 + x2

c.

Maximize
subject to

2
1
3

x1, x2 0

Z = 4x1 + X2 + 2x3
2x1 + x2 + 3x3 10
x1-x2+x3
=4

d. Maximize
subject to

xl,x2,x3 0

Z = 3x1 + 5x2 + 7x3


x1 + x2 + 3x3 10
4x1 x2 + 2x3 15

Minimise
Subject to

x1,x2 0, x3 : unrestricted in sign

Z = x1 + 2x2 + x3
X1 x2 x3
-1
6x1 + 3x2 + 2x3 = 12

Minimize
subject to

Z = 5x1 + 10x2 + 15x3 + 12x4


2x1 + 3x2 + 7x3 + x4
X1 + x2 + x4
-x1 + 4x2 + 5x3

Maximize:

xl,x2,x3 0

50
= 45
30

x1,x2,x3 0, x4 : unrestricted in sign

100X1 + 90X2 + 40X3 + 60 X4


Subject to,
6X1 + 4X2 + 8X3 + 4 X4 140
10X1 + 10X2 + 2X3 + 6X4 120
10X1 + 12x2 + 6X3 + 2X4 50
Xlo X2, X3.X4 2: 0;

(Only formulation is required. Please do not solve.)

Q57

A factory produces three different products, A, B, and C. The profits per unit of these products are Rs. 3,
Rs. 4 and Rs. 6 respectively. The products are processed. in three operations, viz., X, Y and Z and the
time (in hours) required in each operation for each unit is given below:

Operations

X
Y
Z

A
4
5
1

Products
B
C
1
6
3
1
2
3

The factory has 3 machines for operation X, 2 machines for operation Y and only one machine for
operation Z. The factory works 25 days in a month, at the rate of 16 hours a day in two shifts. The
effective working of all the processes is only 80% due to power cuts or breakdown of machines.
(i)
Formulate the problem mathematically.
(ii)
Use simplex method to find how many units of each product should be produced monthly
in order to maximize profit?
(iii)
Write the dual to the above problem and determine the optimum values of the dual variables.
(iv)
What are the shadow prices of the resources?

27

PRAVINN MAHAJAN CLASSES 9871255244, 8800684854

Q58

Noahs Boats make three different kinds of boats. All can be made profitably by the Company, but the
monthly production is constrained by the limited amount of labour, wood and screws available each
month. The Director should choose the combination of boats that maximizes his revenue In view of the
Information given
Input
Labour (hours)
Wood (board feet)
Screws (Kg)
Selling price (Rs)

Rowboat
12
22
2
4,000

Canoe
7
18
4
2,000

Kayak
9
16
3
5,000

Monthly availability
1260 hours
19,008 board feet
396 Kg

1
2
3

Formulate the above as a linear programming problem,


Solve it by the Simplex method,
From the optimal table of the solved LPP, answer the following questions
How many boats of each type will be produced and what will be the resulting
revenue?
Which, If any, of the resources are not fully utillsed? If so, how much of spare
capacity Is left?
How much wood will be used to make all of the boats given in the optimal solution?
State the dual of the formulated LPP,
Q59

Store More Company has three departments - Assembly, Painting and Packing with the capability of
making three types of almirah. An almirah of Type I requires one hour of assembly, 40 minutes of painting
and 20 minutes of packing time respectively. Similarly, Type II requires 80 minutes, 20 minutes and one
hour respectively. The Type III requires 40 minutes each of assembly, painting and packing time. The
total time available at assembly, painting and packing departments are 600 hours, 400 hours and 800
hours respectively.
Determine the number of each type of almirah that should be produced in order to maximize profits. The
unit for types I, II, and III are Rs.40, 80 and 60 respectively
Suppose the Manager is thinking of renting the productive capacities of the three departments to another
manufacturer - Steel Racks Company. Steel Racks Is Interested In minimizing the rental charges. On the other
hand the Store More Company would like to know the worth of a productive hour to them, in each of the
departments determine the rental rates. Formulate the problem as a LPP. Explain clearly M90

Q60

Given below are the objective function, the constraints and the final simplex tableau for a linear
programming product mix problem:
Maximize
Subject to

Z = 2x1 + 5x2 + 8x3


6x1 + 8x2 + 4x3 96
2x1 + x2 + 2x3 40
5x1 + 3x2 + 2x3 60

x1,x2,x3 0

Final simplex tableau


____________________________________________________________________________________
Cj
Product
2
5
8
0
0
0
Quantity
Mix
X1
x2
x3
s1
s2
s3
____________________________________________________________________________________
5

x2

1/3

1/6

-1/3

8/3

x3

5/6

-1/12

2/3

56/3

0
s3
7/3
0
0
-1/3
-1/3
1
44/3
____________________________________________________________________________________
Zj
25/3
5
8
1/6
11/3
0
162 -2/3
Cj Zj
-19/3 0
0
-1/6
-11/3 0
____________________________________________________________________________________

28

PRAVINN MAHAJAN CLASSES 9871255244, 8800684854

(i)
(ii)
(iii)
(iv)
(v)
(vi)
(vii)
Q61

Write the optimum product mix and the profit contribution shown by the above solution.
Is this solution feasible? Why? give brief reason.
Does the problem have any alternative optimum solution? If so, find the other solution.
What are the shadow prices of three departments.
If the company wishes to expand the production capacity, which of the three depart ments should
be given priority.
If the company produces 6 units of Xl, how many units of X2 andx3 shall have to be
reduced, if any?
If a customer is prepared to pay higher prices for product Xl, how much should the price be
increased so that the company's profit remains unchanged?
_____________________________________________________________________________
X1
X2
S1
S2
A1
A2
Basis

Cj

15

25

-M

-M

A1

-M

-1

20

S2

30

A2
-M
3
-2
0
0
9
1
18
_____________________________________________________________________________
Zj

-10M

-4M

-M

-M

-38M

Cj-Zj
15+10M
25+4M -M
0
0
0
____________________________________________________________________________
write down the original primal problem represented by the above tableau.
Find out the optimal solution of this problem. Is it a unique solution? Why?
Write the dual of the problem and determine the optimal solution of the dual
Q62

The Simplex table for a maximization problem of linear programming problem is given below:
________________________________________________________________________________
Cj
Product
4
5
0
0
Quantity
Mix
X1
x2
s1
s2
________________________________________________________________________________
5

x2

10

0
s2
1
0
-1
1
3
_________________________________________________________________________________
Zj
5
5
5
0
50
Cj Zj
-1
0
-5
0
Answer the following questions, giving brief reasons
1.
2.
3.
4.

5.
6.
7.

Is the above solution optimal?


Are there more than one optimal solution?
Is this solution degenerate and is the solution feasible?
If sl is the slack in Machine A (in hours/week) and s2 is the slack in Machine B (in hours / week),
which of these machines is being used to the full capacity when producing according to this
solution?
A customer would like to have one unit of product xl and is willing to pay in excess of the normal
price to get it. How much should the price be increased in order to ensure no reduction in profit?
Machine B (associated with slack s2 in hours/week) has to be shut down for repairs for 2 hours.
next week. What will be the effect on profits?
How many units of the products xl and x2 are being produced as per this solution and what is the
total profit?

29

PRAVINN MAHAJAN CLASSES 9871255244, 8800684854

Q63

The costs and selling prices per unit of two 'products manufacturing by a company are as under:
Product

A (Rs)

B(Rs)

Selling Price
500
450
Variable costs
Direct Materials @ 25 per kg
100
100
Direct Labour @ 20 per kg
80
40
Painting @ Rs. 30 per hour
30
190
Variable Overheads:
190
175
Fixed costs @ Rs. 17.50/Direct labour hour
70
35
Total costs
470
410
Profit .
30
40
In any month the maximum availability of input is limited to the following:
Direct Materials
480 kgs
Direct Labour hours
400 hours
Painting hours
200 hours
Required:
(i)
Formulate a linear programme to determine the production plan which maximjzes the profits
by using graphical approach.
.
(ii)
State the optimal product mix and the monthly profit derived from your solution in (i) above.
(iii)
If the company can sell the painting time as Rs. 40 per hour as a separate service, show
what the modification will be required in the formulation of the linear programming problem.
You are required to re-formulate the problem but not to solve.
N08
Q64

An oil refinery can blend three grades of crude oil to produce quality A and 4 quality B petrol. Two
possible blending processes are available. For each production run, the older process uses 5 units of
crude Q, 7 units of crude P and 2 units of crude R and produces 9 units of A and 7 units of 8. The newer
process uses 3 units of crude Q, 9 units of crude P and 4 units of crude R to produce 5 units of A
and 9 units of B.
..
.
Because of prior .contract commitments, the refinery must produce at least 500 units of A and at least
300 units of B for the next month. It has 1,500 units of crude Q, 1,900 units of crude P and 1,000 units of
crude R. For' each unit of A, refinery receives Rs. 60 while for each unit of B it receives Rs. 90.
Formulate the problem as linear programming model so as to maximise the 'revenue.
N09

Q65

A firm produces three products A,B & C. Its uses two types of raw materials I & II of which 5000 and 7500
units respectively are available. The raw material requirements per unit of product are given below:
Raw Material
I
II

Requirement per unit of product


A
B
C
3
4
5
5
3
5

The labour time for each unit of product A is twice that of product B and three times that of product c. The
entire labour force of the firm can produce the equivalent of 3000 units of A. The minimum demand of the
three products is 600, 650 and 500 units respectively. Also, the ratios of the number of units must be
equal to 2:3:4. Assuming the profits per unit of A,B & C as Rs. 50, 50 and 80 respectively. .
Formulate the problem as linear programming model in order to determine the number of units of each
product which will maximize the profit
N97

30

PRAVINN MAHAJAN CLASSES 9871255244, 8800684854

Q66

A refinery makes three grades of petrol A, B, C from 3 crude oils d, e, f. Crude oil f can be used in any
grade but the others must satisfy the following specifications:
Grade

Selling price per liter

Specification

18.0

Not less than 50% crude d


Not more than 25% crude e

16.5

Not less than 25% crude d


Not more than 50% crude e

15.5

No specifications

There are capacity limitations on the amounts of the 3 crude elements that can be used
Crude
Capacity (Kl)
Price per liter
d
500
19.5
e
500
14.5
f
360
15.1
It is desired to obtain maximum profit. Formulate this as a LPP
Q67

XYZ company has three departments - Assembly. painting and packing with the capability of making
three types of almirahs. An almiras of Type 1 requires one hour of assembly. 40 minutes of painting and
20 minutes of packing time respectively. Similarly. almirah of type II needs 80 minutes of assembly, 20
minutes of painting and one hour of packing time respectively. The last type requires 40 minutes each of
assembly, painting and packing time. The total times available at assembly, painting, and packirig
departments are 600 hours, 400 hours, and 800 hours respectively. Determine the number of each type
of almirahs that should be produced in order to maximize the profit. The unit profits for types I, II and III
are Rs. 40, 80 and 60 respectively.
Suppose the manager is thinking of renting the productive capacities of the three departments to another
almirah manufacturer - Steel Racks Company. Steel Racks is. interested in minimizing the rental charges. On
the other hand the Z company would like to know the worth of a productive hour to them, in each of the
departments to determine the rental rates. Formulate the problem as a LPP. Explain clearly. May/90

Q68

Three grades of Coal A, B and C contains phosphorus and ash as impurities. In a particular industrial
process, fuel upto 100 ton (maximum) is required, which could contain ash not more than 3% and
phosphorus not more than 0.03%. It is desired to maximize the profit while satisfying these conditions.
There is an unlimited supply of each grade. The percentage of impurities and profits of each grade are as
follows
N05
Coal
Phosphorus(%)
Ash (%)
Profit (Rs per ton)
A
0.02%
3.0%
Rs 12.00
B
0.04%
2.0%
Rs 15.00
C
0.03%
5.0%
Rs 14.00
Formulate the LP model to solve it using Simplex Method to determine optimal product mix and profit.

31

PRAVINN MAHAJAN CLASSES 9871255244, 8800684854

Q69

A gear manufacturing company makes two types of gears A and B. Both gears are processed on 3
machines, Hobbing M/c, Shaping M/c and Grinding M/c. The time required by each gear and total time
available per week on each M/c is as follows:
M07 compiler 363
Gear (A)
Gear (B)
Available
Machine
(Hours)
(Hours)
(Hours)
Hobbing M/c
3
3
36
Shaping M/c
5
2
60
Grinding M/c
2
6
60
Other data:
Selling price (Rs.)
820
960
Variable cost (Rs.)
780
900
Determine the optimum production plan and the maximum contribution for the next week by simplex
method. The initial table is given below:
_________________________________________________________________________________
Cj
40
60
0
0
0
Qty
_________________________________________________________________________________
Cj
Variable
x1
x2
x3
x4
x5
0
x3
36
3
3
1
0
0
0
x4
60
5
2
0
1
0
0
x5
60
2
6
0
0
1
__________________________________________________________________________________

Q70

The following matrix gives the unit cost of transporting a product from production plants P1 P2 & P3 to
destinations D1, D2 and D3. Plants P1, P2 and P3 have a maximum production of 65, 24 and 111 units
respectively and destinations 01, O2 and 03 must receive at least 60, 65 and 75 units respectively.
__________________________________________________________________________________
To
D1
D2
D3
Supply
From
__________________________________________________________________________________
P1
400
600
800
65
P2
1000
1200
1400
24
P3
500
900
700
111
Demand
60
65
75
200
___________________________________________________________________________________
You are required to formulate the above as LPP

N08

32

PRAVINN MAHAJAN CLASSES 9871255244, 8800684854

Q71

Transport Ltd. provides tourist vehicles of 3 types- 20-seater vans, 8-seater big cars & 5-seater small
cars. These seating capacities are excluding the drivers. The company has 4 vehicles of the 20-seater
van type, 10 vehicles of the eight-seater big car types 20 vehicles of the 5-seater small car types. These
vehicles have to be used to transport employees of their client company from their residences to their
offices and back. All the residences are in the same housing colony. The offices are at two different
places, one is the Head Office and the other is the Branch. Each vehicle plies only one round trip per day,
if residence to office in the morning and office to residence in the evening. Each day, 180 officials need to
be transported in Route I (from residence to Head Office & back) and 40 officials need to be transported
in Route II (from Residence to Branch office & back). The cost per round trip for each type of vehicle
along each route is given below.
_______________________________________________________________________________________
Rs per round trip per customer
20 seater vans
8 seater big cars
5 seater small cars
______________________________________________________________________________________
Route 1:
Residence:

Head office and back

30

50

60

Route II:
Residence:
Branch office and back
25
40
40
_______________________________________________________________________________________
You are required to formulate the information as a linear programming problem, with the objective of
minimizing the total cost of hiring vehicles for the client company, subject to the constraints mentioned above.
********

33

PRAVINN MAHAJAN CLASSES 9871255244, 8800684854

TRANSPORTATION

The transportation problem is concerned with the allocation of items between suppliers (called origins)
and consumers (called destinations) so that the total cost of the allocation is minimised. The problem can
be solved using either linear programming methods or the special transportation algorithm. Transportation
problems are basically allocation models. The objective is to minimise the cost of transportation of
homogeneous commodity from different supply points to different demand points.
Application
A few applications of the transportation method are mentioned below:
Minimise shipping costs from factories to warehouses (or from warehouses to retail outlets)
Determine lowest cost location for new factory, warehouse, office or other facility.
Find minimum cost production schedule that satisfies firm's demand and production limitations
(called 'production smoothing)
Conditions to use transportation algorithm
To use the transportation algorithm the following conditions must be satisfied:
The cost per item for each combination of origin and destination must be specified.
The supply of items at each origin must be known.
The requirement of items at each destination must be known.
The total supply must equal the total demand.
Q

Explain the steps to solve transportation problem.


Step 1 Find Initial Basic Solution (for balanced minimization problem) using any of the following 3
methods
Points to be noted before ascertaining initial basic solution
Transportation problem should be a balanced problem i.e. demand should
be equal to supply, or availability should be equal to requirements. If the
problem is not balanced, add a dummy row or column
If the transportation problem is a maximization problem, convert the profit
matrix into Loss matrix. This is done by subtracting each entry in the table
from the largest No. in the table.
If it is a restrictive transportation problem with prohibited routes, then assign
M as a maximum cost to the respective cell.
If it is a committed transportation problem i.e. a minimum or maximum no. of
units are to be transferred from one factory to a specified depot, then reduce
the demand and supply of respective depot and factory. But after attaining
initial basic solution allocate that minimum or maximum units to a respective
cell

34

PRAVINN MAHAJAN CLASSES 9871255244, 8800684854

1. Northwest Comer Method


Allocation always starts from upper left hand corner (Le. North side first row-first column).
Assignment is made in such a way that the resources available are exhausted or demand is fully
satisfied.
If the resources available are fully exhausted then we move down to second next row and
continue the process till the whole demand is exhausted.
If the first allocation completely satisfies the demand then we move to the next column of the
same row, and continue the allocation process till all availability and requirements are met.
The procedure is repeated till all row availability and column requirements are met.

2. Least cost method.


Allocation always starts from the cell whose transportation cost per unit is least.
The lowest cell is filled as much as possible in .view of the availability and destination
requirement of its column.
Then we move to the next lowest cell and so on continue the procedure in view of the remaining
availability of demand and supply.
The procedure is repeated till all row availability and column requirements are met.
In case of tie for the lowest cell during allocation, choice may be made for a row or column by
which maximum requirement is exhausted.
3. Vogels Approximation method
Find the difference between two least cost cells in every row and column.
Identify the' row or' column with the highest of the difference. It is in this row or column
where allocation should be made.
In the row or column selected in step 2, identify the least cost cell. It is in this cell
allocation should be made.
If there is a tie amongst the largest differences, the choice may be made for a row or
column which has least cost. In case there is a tie in cost cell also, choice may be made
for a row or column by which maximum requirement is exhausted.
Hatch that row or column containing this cell whose totals have been exhausted so that
this row or column is ignored in further consideration.
Re-compute the row & column differences for the reduced TLP 'go ahead to Step 3.
Continue the above steps till all allocations are made.

35

PRAVINN MAHAJAN CLASSES 9871255244, 8800684854

Step 2 Ascertain if initial basic solution is feasible or not

Initial basic solution is feasible if number of occupied cells = m + n 1, where m = no. of rows
and n = No. of columns
If no. of occupied cells m + n 1 , then initial basic solution is degenerate
To remove such degeneracy a very small quantity (epsilon) is allocated to one or more least
cost independent unoccupied cell ( such least cost cell should be independent means that a
closed path cannot be made from this cell.). If the least cost cell is not independent then next
least cost independent cell is chosen ( can be added to unoccupied dummy cell )
Degeneracy may also arise at later stage. Thus after every improved solution degeneracy is to be
checked ( may have to be used in improved solution also)

Step 3 Check if initial basic feasible solution is optimal or not


It can be done by Modified distribution method (MODI) or Algebra method.
let the row wise costs are Ui &column wise costs are Vj, where
i = 1,2,3.m & j = 1,2,3n
Assign 0 value arbitrarily to a row or column variable Uj or Vj. (assign 0 to U1)
Taking each cost cell (Uj+Vj) = Cij , calculate individually all values of Ui & Vj
Calculate opportunity cost for each unallocated cell i.e. Ui + Vj Cij = ij
If all ij values are 0 or ve, solution is optimal, if any ij value is +ve then solution is not optimal.
If any ij value is 0 it means alternate optimal solution also exist
If solution is not optimal, the cell with largest +ve opportunity cost ij should be selected. Form
this cell a closed loop starting and ending at this cell should be made and reallocate the solution

36

PRAVINN MAHAJAN CLASSES 9871255244, 8800684854

Q1

Plant
A
B
C

Transportation cost (Rs/unit)


Distribution centers
X
Y
50
30
15
27
25
25

Z
20
40
45

Demand

10,000

25,000

15,000

Availabilty (units)
20,000
18,000
12,000

Suggest optimal solution for the following transportation problem and indicate the total minimum
transportation cost
(no degeneracy, optimal sol)
If the company wants atleast 5,000 units to be transported from plant B to distribution center Z,
what will be the transportation schedule and effective cost?
(restriction, no degeneracy, not optimal)
Q2

A product is manufactured by four factories A, B, C and D. The Unit production costs are Rs.2,
Rs.3, Re.1 and Rs.5 respectively. Their daily production capacities are 50, 70, 30 and 50 units
respectively. These factories supply the product to four P, Q, R and S. The demand made by these
stores are 25, 35, 105 and 20 Units transportation cost in rupees from each factory to each store is
given in the following table;
Stores
P
Q
R
S
A
2
4
6
11
B
10
8
7
5
C
13
3
9
12
D
4
6
8
3
Determine the extent of deliveries from each of the factories to each of the stores so that the total
cost (production and transportation together ) is minimum.
(May 2002)
(only table imp, non degeneracy, optimal)

Q3

A compressed Natural Gas (CNG) company has three plants producing gas and four outlets. The
cost of transporting gas from different production plants to the outlets, production capacity of each
plant and requirement at different outlets is shown in the following cost-matrix table:
outlets
Plants
A
B
C
D
capacity of production
X
4
6
8
6
700
Y
3
5
2
5
400
Z
3
9
6
5
600
Requirement 400
450
350
500
1700
Determine a transportation schedule so that the cost is minimized.
The cost in the cost-matrix is given in thousand of rupees.
(Nov 2001)
(Simple, non deg, optimal, multiple optimal)

37

PRAVINN MAHAJAN CLASSES 9871255244, 8800684854

Q4

Solve the following problem using transportation method, obtaining the initial feasible solution by VAM.
The cell entries in the table are unit costs
To
1
2
3
4
5
supply
From
1
80
69
103
64
61
12
2
47
100
72
65
40
16
3
16
103
87
36
94
20
4
86
15
57
19
25
8
5
27
20
72
94
19
8
Demand
16
14
18
6
10
(Degeneracy, non optimal, balanced, difficult path)

Q5

A company has 4 terminals U,V,W and X. At the start of a particular day 10,4,6 and 5 trailers respectively
are available at these terminals. During the previous night 13,10,6 and 6 trailers respectively were loaded
at plants A,B,C,D. The company dispatcher has come up with the costs between the terminals and plants
as follows
Plants
A
B
C
D
U
20
36
10
28
Terminals
V
40
20
45
20
W
75
35
45
50
X
30
35
40
25
Find the allocation of loaded trailers from plants to terminals in order to minimize transportation cost.

Q6

Garg and Garg a leading firm has 3 auditors. Each auditor can work upto 160 hours during the next month,
during which time 3 projects must be completed. Project 1 will take 130 hours, Project 2 will take 140 hours and
project 3 will take 130 hrs. The amount per hour billed for assigning the auditor a project is:
Project (amount in rupees)
1
2
3
Auditor
A
1200 1500 1900
B
1400 1300 1200
C
1600 1400 1500
Formulate this as a transportation problem and find the optimal solution. Also find out the maximum total
billings during the next month

Q7

Consider the following transportation cost table. The costs are given in rupees, supply and demand are in
units. Determine an optimal solution
Destination
1
2
3
4
5
Supply
Source
I
40
36
26
38
30
160
II
38
28
34
34
198
280
III
36
38
24
28
30
240
Demand
160
160
200
120
240
n88
(unbalanced, degeneracy, non optimal, difficult path, two e)

38

PRAVINN MAHAJAN CLASSES 9871255244, 8800684854

Q8

A company has two factories at A, B, and C which supply warehouses at D, E, F and G. Monthly factory
capacities are 160, 150, and 190 units respectively. Monthly warehouse requirements are 80, 90, 110 and
160 units respectively. Unit shipping costs (in rupees) are as follows:
To
D
E
F
G
From
A
42
48
38
37
B
40
49
52
51
C
39
38
40
43
Determine the optimum distribution for this company to minimize shipping costs.
(degen, unbal, min, )

Q9

The cost per unit of transporting goods from factories X, Y, Z to destinations A, Band C and the quantities
demanded and supplied are tabulated below. As the company is working out the optimium logistics, the
company has announced a fall in the oil prices. The revised unit costs are exactly half the costs given in
the table. You are required to evaluate the minimum transportation costs.
Destination
A
B
C
Supply
Factories
X
15
9
6
10
Y
21
12
6
10
Z
6
18
9
10
Demand
10
10
10
30
J09
(Min,bal,deg,2e)

Q10

The initial allocation of a transportation problem along with the unit cost of transportation from each origin
to the destination is given below. You are required to arrive at the minimum transportation cost by the
Vogels approximation method and check for optimality (Hint take u1 = 0 at Row 1 for initial cell valuation)
Requirement
11
9

10

7
9
Availability

12

12

10

11

6
8

18

40

(degeneracy, optimal)

39

M07

PRAVINN MAHAJAN CLASSES 9871255244, 8800684854

Q11

Alpha Company has 3 plants and 3 warehouses. The cost of sending a unit from different plants to the
warehouses, production at different plants and demand at different warehouses are shown in the
following matrix:
Warehouses
A
B
C
Production
Plant
X
8
16
16
152
Y
32
48
32
164
Z
16
32
48
154
Demand
144
204
82
Determine the transportation schedule so that cost is minimized. Assume that cost in the cost matrix is
given in thousands of rupees.
M01
(Min, unbal,tie,no deg, optimal,multi)

Q12

Solve the following transportation problem for minimum cost:


Destination/origin
A
B
C
1
7
4
3
2
3
2
7
3
4
4
3
4
9
7
5
Availability
12
8
35
(Min,unbal,deg,multi)

Q13

D
4
5
7
3
25

Requirement
15
25
20
40
n85

Home building construction company is interested in taking loans from banks for its projects- P,Q,R,S,T.
The rates of interest and the lending capacity differ from bank to bank. All these projects are to be
completed. The relevant details are provided below .
Source bank
P
20
16
15
200

Private bank
Nationalized bank
Co-operative bank
Amount required (in 000s)

Interest rate in % for projects


Q
R
S
T
18
18
17
17
16
16
15
16
15
15
13
14
150
200 125
75

Max credit
(in 000s)
Any amount
400
250

Assuming the role of a consultant, advise the company as to how it should take the loans so that the total
interest payable is least. Find out the alternate optimal solutions, if any. N90 (very multi, min, bal, no deg)
Q14

Solve the following transportation problem and state whether the solution derived by you is unique.
Godown
Factory 1
Factory 2
Factory 3
Factory 4
Demand
N89, big M,min, multi

1
7
9
11
9
60

2
5
11
10
10
20

3
7
6
6
9
40

4
7
11
2
6
20

40

5
5
2
9
40

6
3
5
8
12
40

Stock available
60
20
90
50

PRAVINN MAHAJAN CLASSES 9871255244, 8800684854

Q15

Given the following transportation problem:


Market
A
B
C
Supply
Warehouse
1
10
12
7
180
2
14
11
6
100
3
9
5
13
160
4
11
7
9
120
Demand
240
200
220
It is known that currently nothing can be sent from warehouse 1 to market A and from warehouse 3 to market C.
Solve the problem and determine the least cost transportation schedule. Is the optimal solution obtained unique,
if not what are the other optimal solutions
,(min.bal,non deg,multi, M)

Q16

Timely and Co, a manufacturer must produce a product in sufficient quantity to meet contractual sales in the
next four months. The production capacity and unit cost of production vary from month to month. The
production produced in one month may be held for sale in later months but an estimated storage costs of Re 1
per unit per month. No storage cost is incurred for goods sold in the same month in which they are produced.
There is no opening inventory and none is desired at the end of four months. The necessary details are given
below:
Month
Contracted sales
Maximum production
Unit cost of production
1
20
40
14
2
30
50
16
3
50
30
15
4
40
50
17
How much should the manufacturer produce each month to minimize total cost?
RTP
, M,min,
Consider the following data for the transportation problem:

Q17

Factory

Destination
Supply to be
exhausted
1
2
3
A
5
1
7
10
B
6
4
6
80
C
3
2
5
15
Demand
75
20
50
Since there is not enough supply, some of the demands at the three destinations may not be satisfied.
For the unsatisfied demands, let the penalty costs be rupees 1, 2 and 3 for destinations (1), (2) and (3)
respectively.
M98
Compiler 331,
Q18

Following is the profit matrix based on four factories and three sales depots of the company:
Sales depos
S1
S2
S3
Availability
F1
6
6
1
10
Factories
F2
-2
-2
-4
150
F3
3
2
2
50
F4
8
5
3
100
Requirement
80
120
15
Determine the most profitable distribution schedule and the corresponding profit, assuming no
profit in case of surplus production.
Compiler 320 (max,unbal,nondeg,non optimal)

41

PRAVINN MAHAJAN CLASSES 9871255244, 8800684854

Q19

A Company has four factories F1, F2, F3 and F4, manufacturing the same product. Production and
raw material costs differ from factory to factory and are given in the first two rows of the following
table. The Transportation costs from the factories to sales depots S1, S2 and S3 are given in the
next three rows of the table. The production capacity of each factory is given in the last row.
The last two columns in the table given the sales price and the total requirement at each depot:
Item per
Factory
Sales price
Requirement
unit
Per unit
F1
F2
F3
F4
Production cost
15
18
14
13
Raw material cost
10
9
12
9
80
Transportation cost 3
9
5
4
34
120
1
7
4
5
32
50
5
8
3
6
31
Production capacity 10
150
50
100
Determine the most profitable production and distribution schedule and the corresponding profit.
The surplus should be taken to yield zero profit.
N00, max,unbal,multi compiler 311,

Q20

Q21

A company produces a small component for all industrial products and distributes it to five
wholesalers at a fixed prices of Rs.2.50 per unit. Sales forecasts indicate that monthly deliveries
will be 3,000, 3,000, 10,000, 5,000 and 4,000 units to wholesalers 1,2,3,4 and 5 respectively. The
monthly production capabilities are 5,000, 10,000, 12,500 at plants 1, 2 and 3 respectively. The
direct costs of production of each unit are Rs.1.00 and Rs.0.80 at plants 1, 2 and 3 respectively.
The transportation costs of shipping a unit from a plant to a wholesaler are given below:
1
2
3
4
5
1
0.05
0.07
0.10
0.15
0.15
Plant 2
0.08
0.06
0.09
0.12
0.14
3
0.10
0.09
0.08
0.10
0.15
Find how many components each plant supplies to each wholesaler in order to maximize profit.

(May 2000)

A company has 3 factories and 4 customers. It furnishes the following schedule of profit per unit on
transportation of goods to customers in rupees. You are required to solve the transportation problem to
maximize the profit. Determine the resultant optimal profit
Factory/customer
A
B
C
D
Supply
P
40
25
22
33
100
Q
44
35
30
30
30
R
38
38
28
30
70
Demand
40
20
60
30
m03,

42

PRAVINN MAHAJAN CLASSES 9871255244, 8800684854

Q22

ABC enterprises is having 3 plants manufacturing dry-cells, located at different locations.Production cost differ
from plant to plant. There are 5 sale offices of the company located in different regions of the country. The sales
prices can differ from region to region. The shipping cost from each plant to each sales office and other data are
given by following table:
Production data table
Production cost per unit
Maximum capacity in No. Of units
Plant No
20
150
1
22
200
2
18
125
3

Plant 1
Plant 2
Plant 3

Sales office1
1
9
4

Shipping cost
sales office 2
1
7
5

sales office 3
5
8
3

sales office 4
9
3
2

Demand and sales prices


Sales office1 sales office 2 sales office 3 sales office 4
Demand
80
100
75
45
Sales price
30
32
31
34
Find the production and distribution schedule most profitable to the company.

sales office 5
4
6
7

sales office 5
125
29
N98

Q23

A company has 3 warehouses W1, W2, W3. It is required to deliver a product from these warehouses to 3
customers A,B and C. The warehouses have the following units in stock.
Warehouse
W1
W2
W3
No. of units
65
42
43
Customer requirements
Customer
A
B
C
No. of units
70
30
50
The table below shows the cost of transporting one unit from warehouse to the customer:
Warehouse
W1
W2
W3
A
5
7
8
Customer
B
4
4
6
C
6
7
7
Find the optimal transportation route.
M99

Q24

The manufacturer of jeans is interested in developing an advertisement campaign that will reach four different:
age groups. Advertising campaigns can be conducted through TV, Radio and Magazines. The following table
gives the estimated cost in paise per exposure for each age group according to the medium employed. In
addition, maximum exposure levels possible in each of the media, namely TV, Radio and Magazines are 40, 30
and 20 million respectively. Also the minimum desired exposures within each age group, namely 13-18, 19-25,
26-35 and 36 and older are 30, 25, 15 and 10 millions. The objective is to maximize the cost of attaining the
minimum exposure level in each group
Age group
13-18
19-25
26-35
36 and older
TV
12
7
10
10
Radio
10
9
12
10
Magazines
14
12
9
12
(a)
Formulate the above as a transportation problem, and find the optimum solution.
(b)
Solve this problem if the policy is to provide atleast 4 million exposures through TV in the 13 - 18 age
group, and atleast 8 million exposures through TV in the age group 19 - 25.
M91

43

PRAVINN MAHAJAN CLASSES 9871255244, 8800684854

Q25

A Company wishes to determine an investment strategy for each of the next four years. Five investment types
have been selected, investment capital has been allocated for each of the coming four years, and maximum
investment levels have been established for each investment type. An assumption is that amounts invested
b:: any year will remain invested until the end of the planning horizon of four years. The following table
summarizes the data for this problem. The values in the body of the table represent net return on investment
of one rupee upto the end of the planning horizon. For example, a rupee invested in investment Type B at the
beginning of Year 1 will grow to Rs.l.90 by the end of the fourth year, yielding a net return of Re.0.90.
Investment made at
beginning of Year
1
2
3
4
Max Rupee Invest (000s)

Net Return Data from Investment type


A
B
C
D
E
0.80 0.90
0.60
0.75
1.00
0.55 0.65
0.40
0.60
0.50
0.30 0.25
0.30
0.50
0.20
0.15 0.12
0.25
0.35
0.10
750 600
500
800
1000

Rupees available
(000s)
500
600
750
800

The objective in this problem is to determine the amount to be invested at the beginning of each year in an
investment type, so as to maximize the net rupee return for the four year period. Solve the above
transportation problem and get an optimal solution. Also calculate the net return on investment for the 4 year
planning period.
M93

Q26

The following matrix is a minimization problem for transportation costs. The unit transportation costs are
given at the right hand corner of the cells and the ij values are encircled

D1
F1
F2
F3

D2
3

Demand 300

400

Supply

500

300

4
0

D3

300

Find the optimum solutions and minimum cost

200

1000

M11

44

PRAVINN MAHAJAN CLASSES 9871255244, 8800684854

Q27

A company has 3 plants located at A, B and C. The production of these plants is absorbed by 4
distribution centres located at X, Y , W and Z. The transportation cost per unit has been shown in small
cells in the following table:
Distribution
Centers

Supply
. (units)

Factories
6

13

10

11

6000

14

6000

4000

4000

4500

5000

6000

18000
Demand (units)

17500

Find the optimal solution to the transportation problem by applying Vogels approximation method. N10
Q28

Goods manufactured at 3 plants A, B and C are required to be transported to sales outlets X, Y and
Z.The unit costs of transporting the goods from the plants to the outlets are given below:
Sales outlets / Plants
A
B
C
Total Demand
X
3
9
6
20
Y
4
4
6
40
Z
8
3
5
60
Total supply
40
50
30
120
You are required to:
(i)
Compute the initial allocation by North West corner rule
(ii)
Compute the initial allocation by VAM and check whether it is optional.
(iii)
State your analysis on the optionality of allocation under NWC rule and VAM
M08

Q29

Solve the following transportation problem:


Plant/ Market
M1
M2
P1
10
5
P2
6
4
P3
9
12
Demand
110
80
Show initial solution by
i.
North west corner rule
ii.
Least cost rule
iii.
VAM

M3
7
8
10
190

45

M4
8
5
7
120

Supply
150
125
225
500

PRAVINN MAHAJAN CLASSES 9871255244, 8800684854

Q30

Determine the optimal solution to the problem under VAM


To market
_____________________________________________
M1
M2
M3
M4
_____________________________________________
From
P1
6
4
9
1
Plant
P2
20
6
11
3
P3
7
1
0
14
P4
7
1
12
6
____________________________________________
Demand
90
30
50
30
____________________________________________
Dummy, degeneracy, non optimaldifficult [path

46

Supply
40
40
50
90

PRAVINN MAHAJAN CLASSES 9871255244, 8800684854

Assignment
The assignment problem, is a special case of the linear programming problem. In general items, it is
concerned with a one-to-one assignment. One person to one machine, one machine to one job, etc. The
cost (or profit) of each person-machine, machine-job, or other assignment is known; the objective is to
minimise the total cost or to maximise the total profit of the resource - job assignment.
Hungarian Assignment Method (HAM)
Step 1 Locate the smallest cost element in each row of the cost table. Now subtract this smallest
element from each element in that row. As a result, there shall be at least one zero in each row of
this new table, called the Reduced Cost Table.
Step 2 In the reduced cost table obtained, consider each column and locate the smallest element in it.
Subtract the smallest value from every other entry in the column. As a consequence of this
action, there would be at least one zero in each of the rows and columns of the second reduced
cost table.
Step 3 Draw the minimum number of horizontal and vertical lines (not the diagonal ones) that are
required to cover all the 'zero' elements. If the number of lines drawn is equal to n (the number of
rows/columns) the solution is optimal, and proceed to step 6. If the number of lines drawn is
smaller than n, go to step 4.
Step 4 Select the smallest uncovered (by the lines) cost element. Subtract this element from all
uncovered elements including itself and add this element to each value located at the intersection
of any two lines. The cost elements through which only one line passes remain unaltered.
Step 5 Repeat steps 3 and 4 until an optimal solution is obtained.
Step 6 Given the optimal solution, make the job assignments as indicated by the 'zero' elements. This is
done as follows:
(a)
Locate a row which contains only one 'zero' element. Assign the job corresponding to this
element to its corresponding person. Cross out the zero's, if any, in the column
corresponding to the element, which is indicative of the fact that the particular job and
prson are no more available.
(b)
Repeat (a) for each of such rows which contain only one zero. Similarly, perform the
same operation in respect of each column containing only one 'zero' element, crossing
out the zero(s), if any, in the row in which the element lies.
(c)

If there is no row or column with only a single 'zero' element left, then select a
row/column arbitrarily and choose one of the jobs (or persons) and make the assignment.
Now cross the remaining zeros in the column and row in respect of which the assignment
is made.

(d)

Repeat steps (a) through (c) until all assignments are made.

(e)

Determine the total cost with reference to the original cost table.

47

PRAVINN MAHAJAN CLASSES 9871255244, 8800684854

Q1

A project consist of four (4) major jobs, for which four (4) contractors have submitted tenders. The tender
amounts, in thousands of rupees, are given below.
Jobs
Contractors

120

100

80

90

80

90

110

70

110

140

120

100

90

90

80

90

Find the assignment, which minimizes the total cost of the project. Each contractor has to be assigned
one job.
Compiler 349
M01
Q2

A Production supervisor is considering, how he should assign five jobs that are to be performed, to five
mechanists working under him. He wants to assign the jobs to the mechanists in such a manner that the
aggregate cost to perform the jobs is the least. He has following information about the wages paid to the
mechanists for performing these jobs:
Jobs
Mechanist

10

10

10

Assign the jobs to the mechanists so that the aggregate cost is the least.
Q3

N01

compiler 347

A Marketing Manager has 4 subordinates and 4 tasks. The subordinates differ in efficiency. The tasks also
differ in their intrinsic difficulty. His estimates of the time each subordinate would take to perform each task is
given in the matrix below. How should the task be allocated one to one man so that the total man-hours are
minimized ?
Tasks
I
II
III
IV

Subordinates

1
2
3
4

16
26
76
38

52
56
38
52

34
8
36
48

48

22
52
30
20

Compiler 361

N04

PRAVINN MAHAJAN CLASSES 9871255244, 8800684854

Q4

A BPO company is taking bids for 4 routes in the city to ply pick-up and drop cabs. Four companies have
made bids as detailed below: Min prohibited routes
Compiler 362
Bids for Routes (Rs.)
Company/Routes
R1
R2
R3
R4
C1
4,000
5,000

C2

4,000

4,000
C3
3,000

2,000

C4

4,000
5,000
Each bidder can be assigned only one route. Determine the minimum cost that the BPO should incur. N06

Q5

To stimulate interest and provide an atmosphere for intellectual discussion, a finance faculty in a
management school decides to hold special seminars on four contemporary topics: leasing, portfolio
management, private mutual funds, swaps and options. Such seminars should be held once in a week in the
afternoons. However, scheduling these seminars (one for each topic, and not more than one seminar per
afternoon) has to be done carefully so that the number of students unable to attend is kept to a minimum. A
careful study indicates that the number of students who cannot attend a particular seminar on a specific day is
as follows:
Leasing
Portfolio Management Private Mutual Fund
Swaps & Options
Monday
50
40
60
20
Tuesday
40
30
40
30
Wednesday
60
20
30
20
Thursday
30
30
20
30
Friday
10
20
10
30
Find an optimal schedule of the seminars. Also find out the total number of students who will be missing
at least one seminar.
Unbal min Compiler 357
M99

Q6

Solve the following unbalanced assignment problem of minimizing total time for doing all the jobs. M85
Min unbal pad 16.16
Jobs
I
II
III
IV
V
Operators

Q7

Five workers are available to work with the machine and the respective costs (in Rs) associated with each
worker-machine assignment is given below. A sixth machine is available to replace one of the existing
machines and the associated costs are also stated
unbal min prohib routes
Machines
Workers
M1
M2
M3
M4
M5
M6
A
12
3
6
5
8
B
4
11
5
3
C
8
2
10
9
7
5
D
7
8
6
12
10
E
5
8
9
4
6
RTP

49

PRAVINN MAHAJAN CLASSES 9871255244, 8800684854

Q8

Five swimmers are eligible to compete in a relay team which is to consist of four swimmers swimming four
different swimming styles; back stroke, breast stroke, free style and butterfly. The time taken for the five
swimmers - Anand, Balu, Chandru, Deepak and Eswar - to cover a distance of 100 meters in various
swimming styles are given below in minutes : seconds.' Anand swims the back stroke in 1 : 09, the breast
stroke in 1 : 15 and has never competed in the free style or butterfly. Balu is a free style specialist averaging 1
: 01 for the 100 meters but can also swim the breast stroke in 1 : 16 and butterfly in 1 : 20 Chandru swims all
styles - backstroke 1 :10 breaststroke 1 : 12 free style 1 : 05 and butterfly,1 : 20 Deepak swims only the
butterfly 1 : 11 while Easwar swims the back stroke 1 : 20, the breast stroke 1 : 16, the free style 1 : 06 and
the butterfly 1 : 10. Which swimmers should be assigned to which swimming style? Who will not be in the
Team?
unbal min prohib rout
N91

Q9

The cost of transporting different products to different products to different warehouses of a company is
given. x in the matrix denotes that the particular product is not required in the particular warehouse.
Make optimal assignments of the products and also find the cost of such assignment. Pad 16.13 min
unbal prohib route
Warehouses
I
II
III
IV
V
Product
A
28
19
x
21
22
B
32
25
22
13
18
C
21
18
21
x
17
D
22
29
21
17
28

Q10

Solve the assignment problem represented by the following effective matrix


a
b
c
d
e
f
___________________________________
A
9
22
58
11
19
27

Q11

43

78

72

50

63

48

41

28

91

37

45

33

74

42

27

49

39

32

26

11

57

22

25

18

56

53

31

17

28

min multiple opti sol

N99

ABC airline operating 7 days a week has given the following time-table. Crews must have minimum layover of
5 hours between flights. Obtain the pairing flights that minimize the layover time away from home. For any
given pairing the crew will be based at the city that results in the smaller layover.
M00
Hyderabad-Delhi
________________________________
Flight No.
Departure.
Arrival
A1
6 AM
8 AM
A2
8 AM
10 AM
A3
2 PM
4 PM
A4
8 PM
10 PM

Delhi-Hyderabad
________________________________
Flight No.
Departure.
Arrival
B1
8 AM
10 AM
B2
9 AM
11 AM
B3
2 PM
4 PM
B4
7 PM
9 PM
Compiler 353

50

PRAVINN MAHAJAN CLASSES 9871255244, 8800684854

Q12

WELLDONE Company has taken the third floor of a multistoried building for rent with a view to locate one
of their zonal offices. There are five. main rooms in this floor to be assigned to five managers. Each room
has its own advantages and disadvantages. Some have windows, some are closer to the washrooms or
to the canteen or secretarial pool. The rooms are of all different sizes and shapes. Each of the five
managers was asked to rank their room preferences amongst the rooms 301, 302, 303, 304 and 305.
Their preferences were recorded in a table as indicated below:

M1
302
303
304

M2
302
304
305
301

Manager
M3
303
301
304
305
302

M4
302
305
304
303

M5
301
302
304

Most of the manager did not list all the 5 rooms sice they were not satisfied with some of these Rooms
and they have left off these from the list. Assuming that their preferences can be qualified by numbers,
find out as to which manager would be assigned to which room so that their total preference is minimum.
N90, minimize ranking
Q13

The XYZ co. has 5 jobs A,B,C,D and E to be done and 5 men L,M,N,O and P to do these jobs. The
number of hours each man would take to accomplish each job is given by take to accomplish each job is
given by the following table. Work out the optimum assignment and the total minimum time taken.
Men

Jobs

bal,min,multi

16

13

17

19

20

14

12

13

16

17

14

11

12

17

18

11

10

Q14

Solve the following assignment problem and obtain the minimum cost at which all the jobs can be performed.
Min unbal multi
Job (cost in 00 Rs)
Worker
1
2
3
4
5
A
25
18
32
20
21
B
34
25
21
12
17
C
20
17
20
32
16
D
20
28
20
16
27

Q15

A solicitors firm employs typists on hourly piece rate basis for their daily work. There are 5 typists and their
charges and speed are different. According to an earlier understanding , only one job is given to one typist and
the typist is paid for a full hour even when he works for a fraction of an hour. Find the least cost allocation for
the following data:
Typist
A
B
C
D
E

Rate/hour (Rs) No. of pages typed per hour


5
12
6
14
3
8
4
10
4
11

51

Job
P
Q
R
S
T

No. of pages
199
175
145
298
178 min

PRAVINN MAHAJAN CLASSES 9871255244, 8800684854

Q16

A hospital has to pay Nurses for 40 hours a week. One Nurse is assigned to one patient. The cost per hour for
each of the Nurses is given below:
(a)
Find the Nurse- Patient combination to minimize cost to the hospital.
(b)
How much does each nurse earn per week?
Patient W
Patient X
Patient Y
Nurse K
10
10
30
Nurse L
30
10
20
Nurse M
20
30
20
Suppose that a new patient Z is admitted, and that a new nurse N is appointed. The new patient is charged Rs
40 per hour by each of the existing nurses. The new nurse charges Rs 50 per hour irrespective of the patient.
(c)
(d)

What would be your revised calculations?


Comment on the new solution.

M10

Q17

A city corporation has decided to carry out road repairs on main four arteries of the city. The government
has agreed to make a special grant of Rs 50 lakhs towards the cost with a condition that the repairs must
be done at the lowest cost and quickest time.
If conditions worsens. then a supplementary token grant will also be considered favorably. The
corporation has floated tenders and 5 contractors have sent in their bids. In order to expedite work. one
road will be awarded to only one contractor
Cost of Repairs (Rs lakhs)
Contractors /Road
R1
R2
R3
R4
C1
9
14
19
15
C2
7
17
20
19
C3
9
18
21
18
C4
10
12
18
19
C5
10
15
21
16
(i)
Find the best way of assigning the repair work to the contractors and the costs.
(ii)
If it is necessary to seek supplementary grants, then what should be amount sought?
(iii)
Which of the five contractors will be unsuccessful in his bid?
(iv)
If C1 unable to accept any work, find best assignment

Q18

A company plans to assign 5 salesman to 5 districts in which it operates. Estimates of sales revenue in
thousands of rupees for each salesman in different districts are given in the following table. In your
opinion, what should be the placement of the salesman if the objective is to maximize the expected sales
revenue?
District
D1
D2
D3
D4
Salesman
S1
40
46
48
36
48
S2
48
32
36
29
44
S3
49
35
41
38
45
S4
30
46
49
44
44
S5
37
41
48
43
47
bal max
A firm produces 4 products. There are 4 operators who are capable of producing any of these 4 products.
The firm records 8 hours a day and allows 30 minutes for lunch. The processing time in minutes and profit
for each of the products are given below:
Products
A
B
C
D
Operator
1
15
9
10
6
2
10
6
9
6
3
25
15
15
9
4
15
9
10
10
Profit (Rs) per unit
8
6
5
4
Find the optimal assignment of products to operators
bal max,

Q19

52

PRAVINN MAHAJAN CLASSES 9871255244, 8800684854

Q20

A company has four zones open and four marketing managers available for assignment. The zones are
not equal in sales potentials. It is estimated that a typical marketing manager operating in each zone
would bring in the following Annual sales:
Zones
Rs.
East
2,40,000
West
1,92,000
North
1,44,000
South
1,20,000
The four marketing manages are also different in ability. It is estimated that working under the same
conditions, their yearly sales would be proportionately as under:
Manager M
:
8
Manager N
:
7
Manager O
:
5
Manager P
:
4
Required:
If the criterion is maximum expected total sales, find the optimum assignment and the maximum sales. N07
Compiler 365

Q21

The Marketing director of a multi unit company, Mr Ramesh Arora is faced with the problem of assigning 5
senior marketing managers to 6 zones. The efficiency table of these managers is given belowZones
I
II
III
IV
V
VI
Managers
A
75
93
89
84
80
32
B
83
87
71
78
76
87
C
77
74
85
86
80
93
D
95
98
88
93
85
84
E
92
93
81
91
71
78
As an advisor to the company, recommend which zone should be managed by a junior manager because of
non availability of one more senior marketing manager, so as to maximize the efficiency of the company.
Max,unbal

Q22

Solve the following assignment problem to maximize the production.


Machines
A
B
C
D
Operators
1
10
5
7
8
2
11
4
9
10
3
8
4
9
7
4
7
5
6
4
5
8
9
7
5

Q23

max unbal

An airline has drawn up a new flight schedule involving 5 flights. To assist in allocating 5 pilots to the flights, it
has asked them to state their preference scorers by giving each flight a number out of 10. The higher the
number, greater the preference is. Certain of these flights are unsuitable to some pilots owing to domestic
reasons. These have been marked with a x.
max,bal,prohib
Flight Number
F1
F2
F3
F4
F5
Pilots
A
8
2
x
5
4
B
10
9
2
8
4
C
5
4
9
6
x
D
3
6
2
8
7
E
5
6
10
4
3
What should be the allocation of pilots to flights in order to meet as many preferences as possible.

53

PRAVINN MAHAJAN CLASSES 9871255244, 8800684854

Q24

A methods engineer wants to assign 4 new methods to 3 work centers. The assignment of the new methods
will increase production, details of which are given below. Determine the optimal assignment, if only one
method can be assigned to each work center.
Increase in production(units)
in work centers
A
B
C
Methods
1
10
7
8
2
8
9
7
3
7
12
6
4
10
10
8
pad 16.5 unbal max multiple

Q25

The captain of the playwell cricket team has to allot 5 middle order batting positions to five batsmen. The
average run scored by each batsmen at these positions are as follows
M92
Batting positions
I
II
III
IV
V
P
40
40
35
25
50
Batsman
Q
42
30
16
25
27
R
50
48
40
60
50
S
20
19
20
18
25
T
58
60
59
55
53
1. Find the assignment of batsmen to positions, which would give the maximum number of runs.
2. What will be the total runs scored if batsmen T wants only the III position
3. If another bats men U with the following average runs in the batting positions as given below:
Batting position
I
II
III
IV
V
Average Runs
45
52
38
50
49
Is added to the team, should he be included to play in the team?, If so who will be replaced by him?

Q26

A manufacturing company has four zones East, West, North and South and four sales engineers A,B,C, and D
respectively for assignment. The zones are not equally rich in sales potential. Therefore, it is estimated that a
particular engineer operating in a particular zone will bring the following sales
East Rs 4,20,000, West Rs 3,36,000,

North Rs 2,94,000, South Rs 4,62,000

The sales engineers have different sales ability, working under the same conditions, their yearly sales are
[proportional to 14, 9, 11 and 8 respectively. The criteria for maximum expected total sales is to be met by
assigning the best sales engineer to the richest zone, the next best to the second richest zone and so on. Find
the optimum assignment and the maximum sales.
M98
Q 27

five star hotel has 4 banquet halls that can be used for all functions including weddings. The halls are all about
the same size but the facilities in each hall differs. During the heavy marriage season, 4 parties approached
the executive director to reserve a hall for wedding to be celebrated on the same day. These marriage parties
were told that the first choice among these four halls would cost Rs 10,000 for the day. They were also
required to indicate the second third and fourth preferences and the price that they would be willing to pay.
Marriage party A and D indicated that they woudnt be interested in halls H3 and H4. Other relevant particulars
are given belowRevenue per hall(Rs)
H1
H2
H3
H4
Marriage party A
10,000 9,000 B
8,000 10,000 8,000 5,000
C
7,000 10,000 6,000 8,000
D
10,000 8,000 Decide on the allocation that will maximize the revenue to the hotel.

54

PRAVINN MAHAJAN CLASSES 9871255244, 8800684854

Q28

A car hiring company has one car at each of the 5 depots A,B,C,D and E. A customer in each of the 5 towns V,
W, X, Y and Z requires a car. The distance in Kms. Between depots (origin) and the towns (destinations) are
given in the following table:
Depots
A
B
C
D
E
V
3
5
10
15
8
W
4
7
15
18
8
X
8
12
20
20
12
Z
10
10
15
25
10
Find out as to which car should be assigned to which customer so that the total distance travelled is a
minimum. How much is the total travelled distance?

Q29

The cost matrix giving selling costs per unit of a product by salesman A,B,C and D in Regions R1, R2, R3
and R4 is given below:
Assign one salesman to one region to minimize selling cost.
If the selling price of the product id Rs 200 per unit and variable cost excluding the selling
cost given in the table is Rs 100 per unit, find the assignment that would maximize the
contribution.
What other conclusions can you make from the above?
A
B
C
D
R1
R2
R3
R4

Q30

4
20
36
52

12
28
44
60

16
32
48
64

8
24
40
56

N08

A firm is contemplating the introduction of three products 1,2 and 3 in the 3 plants A,B and C. Only a
single product is decided to be introduced in each of the three plants. The unit cost of production is given
in the following matrix:
A
8
10
7

1
2
3

B
12
6
6

C
M
4
5

(M indicates
impossible
assignment)

A. How should the products be assigned so that the total unit cost is minimum?
B. If the quantity of different products to be produced is as follows then which assignments shall
minimize the aggregate production cost?
C. It is expected that the selling prices of the products produced by different plants would be
different. Assuming that the the quantities given at (b) above would be produced and sold, how
should the products be assigned to maximize profit?

1
2
3

A
15
18
12

B
18
16
10

C
10
8

55

PRAVINN MAHAJAN CLASSES 9871255244, 8800684854

Q31

A company has 4 territories and 4 salesmen available for assignment. Te territories are not equally rich in their
sales potential. The annual total sales potential in each territory is- Territory I Rs 60,000, Territory II Rs
50,000, Territory III Rs 40,000, Territory IV Rs 30,000.
Four salesman differ in ability. Estimation of their yearly sales proportion under the same working conditions
areSalesman
A
B
C
D
Proportion
7
5
5
4
To maximize expected total sales, the intitutive answer is to assign the best salesman to the richest territory
the next best salesman to the second richest and so on. Verify this answer by assignment technique
RTP

56

PRAVINN MAHAJAN CLASSES 9871255244, 8800684854

SIMULATION

What is simulation and what are the steps in simulation?


SIMULATION means to assume the mere appearance of without the reality. Thus the
appearance is true but not real, which implies that simulation is imitation of reality. Simulation is the
representation of a system by a model which will react to change in a similar way to that which is being
simulated.
Simulation may be defined as "a quantitative procedure which describes a process by developing a model
of that process and later conducting a series of organized trial and error experiments to predict the
behaviours of the process over a period. In other words, simulation portrays how the real process would
react to certain change."
Steps Involved in Simulation Process
The following are the steps involved generally in a Simulation Process.
1.
2.
3.
4.
5.
6.
7.
8.

Identification of the problem or system which is intended to be s!mulated.


Formulation of the model intended to be used.
Testing of the model by comparing its behaviour with the behaviour of the actual problem
environment
Identification and collection of the data required for testing of the model.
Running of the simulation.
Analysing the results of simulation in case necessary, the solution which is being evaluated may
be changed
Re-running of the simulation to test the new solution
Validation of the simulation, Le., the inferences drawn which will be appropriate for running the
simulation valid

Enumerate the advantages and disadvantages of Simulation


Advantages
Simulation can be used to investigate the behaviour of problems which are too complex to be
modelled mathematically.
The technique can also be used when the variables in the problem e.g., arrival time, service time
do not follow the standard distributions required for the mathematical models, i.e., Poisson
distribution, Normal exponential distribution.
The basic principles of the simulation technique are fairly simple and it is, therefore, more
attractive to people who are not expert in quantitative techniques.
Simulation does not interfere with the real world system but only with table model and, therefore
it results in saving of cost.
It is a micro analysis of big and complicated system by breaking into each sub-system and
studying the interface of the various sub-systems.
Time will be saved in simulation e.g.,the effects of ordering, advertising or other policies over
many months or years can be obtained by computer simulation in a short time.
Simulation allows us to study the interactive effect of individual components or variables in order
to determine which are important.

57

PRAVINN MAHAJAN CLASSES 9871255244, 8800684854

Disadvantages
Simulation is not an optimizing technique. It simply allows us to select the best of the alternative
systems examined.
Reliable results are possible only if the simulation is continued for a long period.
A computer is essential to cope with the amount of calculation in simulation modeling.
To develop a simulation model means consumption of voluminous data and it may be very costly.
Each simulation model is unique and its solution cannot be applied to other problems however
similar they may be.
The simulation model does not produce answers by itself. Managers must generate all of the
conditions and constraints for solutions they want to examine.
Simulation methods generally are not as efficient as the analytical methods
Q

What is Monte carlo Simulation and what are the steps in Monte carlo simulation? Also enlist its usage in
inventory control.
Monte Carlo simulation Is a special technique of simulation which Involves the selection of random
observations within the simulation model. It is used to solve problems that depend upon probability,
where physical experimentation is virtually unpractical and it is practically impossible to create
mathematical formula.
It is used to examine Inventory, queing, scheduling and forecasting problems.
Steps:
1.

Determination and measure of effectiveness which may be either of


(I)
Maximisatlon of ROI
(II).
Minimlsation of inventory holding cost, stock etc.

2.

Identification of those variables which Influences the measures of effectiveness significantly.

Determination and cumulative probability distribution of each variable selected under steps

4.

Plotting these cumulative probability distribution with probability on the vertical axis and values of
variables on horizontal axis.

5.

Obtaining a set of random numbers from tables

6.

Now consider each random number as decimal value of the cumulative probability distribution
and enter the cumulative distribution plot from the vertical axis.

7.

Now project this point horizontally, until! it intersects cumulative probabilitydistribution curve. Then
project the point of intersection down into the vertical axis.

8.

Then record the value generated above in the formula derived.

9.

Solve and record the value. This value is the objective for that simulated value.

10.

Steps 6 to 9 are repeated until sample size is large enough to the satisfaction of the decision
maker.

Monte carlo Simulation has following applications in Inventory controla.


b.
c.
d.

Determination of Reorder level and Reorder quantity.


Computation of stock out costs and impact on profit.
Analysis of value of storage facilities for avoding stockouts and impact on profits
Analysis of demand distribution during lead time.

58

PRAVINN MAHAJAN CLASSES 9871255244, 8800684854

Q1

A car rental agency has collected the following Data on the demand for five- seater vehicles over the past
50 days
Daily Demand
No. of days

4
4

5
10

6
16

7
14

8
6

The agency has only 5 cars currently.


i.
Use the following 5 random numbers to generate 5 days of demand for the rental agency.
Random Numbers: 15,48,71,56,90
ii.
What is the average numbers of cars rented per day for 5 days ?
iii.
How many rentals will be lost over the 5 days ?
M11
Q2

ABC Co-operative Bank receives and disburses different amount of cash in each month. The bank has an
opening cash balance of Rs 15 crores in the first month. Pattern of receipts and disbursements from past
data is as follows:
Monthly cash receipts
Monthly Cash disbursements
Rs. In crores
Probability
Rs. In crores
Probability
30
0.20
33
0.15
42
0.40
60
0.20
36
0.25
39
0.40
99
0.15
57
0.25
Simulate the Cash position over a period of 12 months. Required:
1.
Calculate the probability that the ABC Cooperative Bank will fall short in payments.
M10
2.
Calculate the average monthly shortfall.
3.
If ABC Bank can get an overdraft facility of Rs.45 Crores from other nationalized banks, what Is
the probability that they will fall short in Monthly Payments?
Use the following sequence (row-wise) of paired Random Numbers.
1778 4316 7435 3123 7244 4692 5158 6808 9358 5478 9654 0977

Q3

With a view to improving the quantity of customer services, a Bank is interested In making an assessment of
the Waiting time of its Customers coming to of Its branches located in residential area. This branch has only
one telling counter. The arrival rate of the Customers and the Service Rate of Teller are given below:

Time between two consecutive arrivals


of customers
3 minutes
4 minutes
5 minutes
6 minutes
7 minutes

Probability

Service time by the teller

0.17

3 minutes
4 minutes
5 minutes
6 minutes
7 minutes

0.25
0.25
0.20
0.13

Probability
0.10
0.30
0.40
0.15
0.05

You are required to Simulate 10 arrivals of Customers in the system starting from 11 AM and show the waiting
time of the Customers and idle time of the teller. Use the following random numbers taking the first two random
numbers in two digits each for first trial and. so on. 11, 56, 23, 72, 94, 83, 83, 02, 97, 99, 83, 10, 93, 34, 33, 53,
49, 94, 37 and 97.
M10

59

PRAVINN MAHAJAN CLASSES 9871255244, 8800684854

Q4

At a small store of readymade garments, there is one clerk at the counter who is to check bills, receive
payments the packed garments into fancy bags. The arrival of customer at the store is random and
service time varies from one minute to 6 minutes the frequency distribution for which is given below:
Time between arrivals (minutes)
1
2
3
4
5
6

Frequency
5
20
35
25
10
5

Service time (in minutes)


1
2
3
4
5
6

Frequency
1
2
4
2
1
0

The store starts work at 11 a.m. and closes at 12 noon for lunch and the Customers are served on the
"first came ft served basis". Using Monte Carlo Simulation technique, find average length of waiting line,
average waiting time, average service time and total time spent by a customer in system. You are given
the following set of random numbers, first twenty for arrivals and last twenty for service:
N09
64
07
30
11
Q5

04
08
75
79

02
59
38
61

70
53
24
77

03
01
57
10

60
62
09
16

16
36
12
55

18
27
18
52

36
97
65
59

38
86
25
63

A single counter ticket booking centre employs one booking clerk. A passenger on arrival Immediately goes
to the booking counter for being served If the counter is free. If, on the other hand, the counter is engaged,
the passenger will have to wait. The passengers are served on first come first served basis. The time of
arrival and the time of service varies from one minute to six minutes. The distribution of arrival and services
time is as under:
Arrival / service time
1
2
3
4
5
6

Arrival (Probability)
0.05
0.20
0.35
0.25
0.10
0.05

Service (Probability)
0.10
0.20
0.40
0.20
0.10
-

Simulate the arrival and service of 10 passengers starting from 9 A.M by using the following random
numbers in pairs respectively for arrival and services.
Random numbers 60, 09, 16, 12, 08, 18, 36, 65, 38, 25, 07, 11, 08, 79, 59, 61, 53, 77, 03, 10.
Determine the total duration of Idle time of Booking Clerk and Waiting time of passengers
N08
Q6

A Publishing house has bought out a new monthly magazine, which sells at Rs. 37.5 per copy. The cost of
producing it is Rs. 30 per copy. A Newsstand estimates the sales pattern of the magazine as follows:
Demand Copies
Probability
0 < 300
0.18
300 < 600
0.32
600 < 900
0.25
900 < 1200
0.15
1200 < 1500
0.06
1500 < 1800
0.04
The newsstand has contracted for 750 copies of the magazine per month from the publisher. The unsold
copies are returnable to the publisher who will take them back at cost less Rs. 4 per copy for handling
charges.
The newsstand manager wants to simulate of the demand and profitability. The of following
random number may be used for simulation:
27, 15, 56, 17, 98, 71, 51, 32, 62, 83, 96, 69.

60

PRAVINN MAHAJAN CLASSES 9871255244, 8800684854

You are required to(i) Allocate random numbers to the demand patter forecast by the newsstand.
(ii) Simulate twelve months sales and calculate the monthly and annual profit/loss.
(iii) Calculate the loss on lost sales.
Q7

A company trading in motor vehicle spares wishes to determine the level of stock it should carry for the item in
its range. Demand is not certain and replenishment of stock takes 3 days.
For one item X, the following information is obtained:
Demand (unit per day)
Probability
1
.1
2
.2
3
.3
4
.3
5
.1
Each time an order is placed, the company incurs an ordering cost of Rs. 20 per order. The company also
incurs carrying cost of Rs. 2.50 per unit per day. The inventory carrying cost is calculated on the basis of
average stock.
The manager of the company wishes to compare two options for his inventory decision.
(A)
Order 12 units when the inventory at the beginning of the day plus order outstanding is less
than 12 units.
(B)
Order 10 units when the inventory at the beginning of the day plus order outstanding is
less than 10 units.
Currently (on first day) the company has a stock of 17 units. The sequence of random number to be used
is 08, 91, 25, 18,40, 27, 85, 75, 32, 52 using first number for day one.
You are required to carry out a simulation run over a period of 10 days, recommended which option the
manager should chose

Q8

A bakery shop keeps stock of a popular brand of cake. Previous experience indicates the daily demand as
given here:
Daily demand :
0
10
20
30
40
50
Probability :
0.01
0.20
0.15
0.50
0.12
0.02
Consider the following sequence of random numbers;
Random. No. 48, 78, 19, 51, 56, 77, 15, 14, 68, 09
Using this sequence, simulate the demand for the next 10 days. Find out the stock situation if the owner of the
bakery decides to make 30 cakes every day. Also, estimate the daily average demand for the cakes on the
basis of simulated data
N99

Q9

A book-store wishes to carry Systems Analysis and Design in stock. Demand is probabilistic and
replenishment of stock takes 2 days (i.e., if an order is placed in March 1, it will be delivered at the end of the
day on March 3). The probabilities of demand are given below:
Demand (daily) :
0
1
2
3
4
Probability :
0.05
0.10
0.30
0.45
0.10
Each time an order is placed, the store incurs an ordering cost of Rs.10 per order. The store also incurs a
carrying cost of Rs.0.50 per book per day. The inventory carrying cost is calculated on the basis of stock at the
end of each day. The manger of the book-store wishes to compare two options for his inventory decision:
A.
Order 5 books, when the inventory at the beginning of the day plus orders outstanding is
less than 8 books.
B.
Order 8 books, when the inventory at the beginning of the day plus orders outstanding is
less than 8 books.
Currently (beginning of the 1st day) the store has stock of 8 books plus 6 books plus 6 books ordered 2
days ago and expected to arrive next day. Using Monte-Carlo simulation for 10 cycles, recommend which
option the manager should choose?
The two digits random numbers are given below:
89, 34, 78, 63, 81, 39, 16, 13, 73
(M00)

61

PRAVINN MAHAJAN CLASSES 9871255244, 8800684854

Q10

A retailer deals in a perishable commodity. The daily demand and supply are variables. The data for the
past 500 days show the following demand and supply:
Supply
Availability (Kg)
10
20
30
40
50

No. of days
40
50
190
150
70

Demand (Kg)
10
20
30
40
50

Demand
No. of days
50
110
200
100
40

The retailer buys the commodity at Rs.20 per kg and sells it at Rs.30 per kg. Any commodity remains at the end
of the day, has no saleable value. Moreover, the loss (unearned profit) on any unsatisfied demand is Rs.8 per
kg. Given the following pair of random numbers, simulate 6 days sales, demand and profit.
(31, 18); (63, 84); (15, 79); (07, 32) (43, 75); (81, 27)
The first random number in the pair is for supply and the second random number is for demand viz. in the
first pair (31, 18), use 31 to simulate supply and 18 to simulate demand.
N00
Q11

An investment company wants to study the investment projects based on market demand, profit and the
investment required, which are independent of each other. Following probability distributions are estimated for
each of these three factors:
Annual Demand (Units in thousands): 25
30
35
40
45
50
55
Probability
0.05
0.10
0.20
0.30
0.20
0.10
0.05
Profit
Probability

3.00
0.10

5.00
0.20

7.00
0.40

Investment required (in 000s of Rs):


Probability

2,750
0.25

3,000
0.50

3,500
0.25

9.00
0.20

10.00
0.10

Using simulation process, repeat the trial 10 times, compute the investment on each trail taking these factors
into trail. What is the most likely ret
Use the following random numbers : urn ?
(30, 12, 16);
(59, 09, 69);
(63, 94, 26);
(27, 08, 74);
(64, 60, 61);

(28, 28, 72);

(31, 23, 57);

(54, 85, 20);

(64, 68, 18);


(32, 31, 87).
In the bracket above, the first random number is for annual demand, the second one is for profit and the
last one is for the investment required.
M01
Q12

A Car Manufacturing Company manufactures 40 cars per day. The sale of cars depends upon demand which
has the following distribution:
Sales of cars
Probability
37
0.10
38
0.15
39
0.20
40
0.35
41
0.15
42
0.05
The production cost and sale price of each car are Rs.4 lakh and Rs.5 lakh respectively. Any unsold car is to
be disposed off at a loss of Rs.2 lakh per car. There is a penalty of Re.1 lakh per car, if the demand is not met.
Using the following random numbers, estimate total profit/loss for the company for the next ten days:
9, 98, 64, 98, 94, 01, 78, 10, 15, 19
If the company decides to produce 39 cars per day, what will be its impact on profitability?

62

M02

PRAVINN MAHAJAN CLASSES 9871255244, 8800684854

Q13

Param and Karam are workers on a two-station assembly line. The distribution of activity time at their
stations is as follows:
Time (in sec)
Time frequency for param
Time frequency for karam
10
4
4
20
6
5
30
10
6
40
20
7
50
40
10
60
11
8
70
5
6
80
4
4
(i)
Simulate operation of the line for eight times. Use the random numbers given below:

14
01
95
40
(ii)

Operation 1
61
82
00
03

Operation 2
36
76
55
25

97
41
13
34

Assuming Karam must wait until Param completes the first item before starting work. Will he
have to wait to process any of the other eight items? Explain your answer, based upon your
simulation

Q14

A company manufactures around 200 mopeds. Depending upon the, availability of raw materials and other
conditions, the daily production has been varying from 196 mopeds to 204 mopeds, whose probability
distribution is as given. below: '
Production/day:
Probability
196
1.00
197
0.95
198
0.86
199
0.74
200
0.60
201
0.40
202
0.25
203
0.14
204
0.06
The finished mopeds are transported in a specially designed three storied lorry that can accommodate
only 200 mopeds.
Use the following 15 random numbers 82, 89,78,24, 53, 61, 18, 45, 04, 23,50, 77, 27, 54,10 to simulate
the process to find out:
(i)
What will be the average number of mopeds waiting in the factory?
(ii)
What will be the average number of empty space on the lorry?

Q15

Dr. STRONG is a dentist' who schedules all her patients for 30 minutes appointments. Some of the patients
take more or less than 30 minutes depending on the type of dental work to be done. The following summary
shows the various categories of work, their probabilities and the time needed to complete the work:
Category
Time Required
Probability
Filling
45
0.40
Crown
60
0.15
Cleaning
15
0.15
Extraction
45
0.10
Check up
15
0.20
Simulate the dentist's clinic for four hours and determine the average waiting time for the patients as well
as the idleness of the doctor. Assume that all the patients show up at the clinic at exactly their scheduled
arrival time starting at 8.00 A.M. Use the following random numbers handling the above problem:
40
82
11
34
25
66
17.
79
N90

63

PRAVINN MAHAJAN CLASSES 9871255244, 8800684854

Q16

The occurrence of rain in a city on a day is dependent upon whether or not it rained on the previous day.
If it rained on the previous day, the.rain distribution is given by :
Event
Probability

No rain
0.50

1 cm. Rain
0.25

2 cm. Rain
0.15

3 cm. Rain
0.05

4cm.Rain
0.03

5cm.Rain
0.02

If it did not rain the previous day, the rain distribution is given by :
Event
Probability

No rain
0.75

1 cm. Rain
0.15

2 cm. Rain
0.06

3 cm. Rain
0.04

Simulate the city's weather for 10 days and determine by simulation the total days without rain as well as
the total rainfall during the period. Use the following random numbers:
67
63
39
55
29
78
70
06
78
76
for simulation. Assume that for the first day of the simulation it had not rained the day before.
N93
Q17

The output of a production line is checked by an inspector for one or more of three different types of defects,
called defects A, 8 and C. If defect A occurs, the item is scrapped. If defect B or C occurs, the item must be
reworked. The time required to rework a B defect is 15 minutes and the time required to rework a C defect is
30 minutes: The probabilities of an A, B and C defects are 0.15, 0.20 and 0.10 respectively. For ten items
coming off the assembly line, determine the number of items without any defect, the number scrapped, the
total minutes of rework time & total no of good units. Use the following random numbers:
RN for defect A
RN for defect B
RN for defect C

Q18

48
47
82

55
36
95

91
57
18

40
04
96

93
79
20

01
06
04

83
10
56

63
13
11

47
57
52

52
09
03

M04

A book store wishes to carry 'Ramayana' in stock. Demand is probabilistic and replenishment of stock takes 2
days (i.e. if an order is placed on March 1, it will be delivered at the end of the day on March 3). The
probabilities of demand are given below
Demand (daily)
Probability

0
0.05

1
0.10

2
0.30

3
0.45

4
0.10

Each time an order is placed, the store incurs an ordering cost of Rs10 per order. The store also incurs a
carrying cost of Rs 0.50 per book per day. The stock out cost is Rs 5 per unit. The inventory carrying cost is
calculated on the basis of stock at the end of each day.
The manager of the book store wishes to compare two options for his inventory decision.
A.

Order 5 books when the inventory at the beginning of the day plus orders outstanding is less than 8
books.
B.
Order 14 books when the inventory at the beginning of the day plus orders outstanding is less
than 8.
Currently (beginning of 1 st day) the store has a stock of 8 books plus 6 books ordered two days ago and
expected to arrive next day. Using Monte Carlo Simulation for 10 cycles, recommend which options the
manager should choose.
The two digit random nos. are : 89, 34, 78, 63, 61, 81, 93, 16, 13, 73

64

PRAVINN MAHAJAN CLASSES 9871255244, 8800684854

Q19

A dealer in colour TVs wants to use a scientific method for reducing his investment in stocks. The
daily demand for a T.V. is random and varies from day to day in an unpredictable pattern. From the past sales
records, the dealer has been able to establish a probability distribution of the demand as given below:
Daily Demand (units)
2
3
4
5
6
7
8
9
10
Probability
0.06
0.14
0.18
0.17
0.16
0.12
0.08
0.06
0.03
The dealer also known from his past experience that the lead time is almost fixed at 5 days. The dealer
would like to study the implications of a possible inventory policy of ordering 30 units, whenever the
inventory at the end of the day is 20 units. The inventory on hand is 30 units & use the following
sequence of random numbers to simulate the demand for next 25 days.
Random Numbers: 3, 38,17,32,69,24,61,30,3,48,88,71,27,80,33,90,78,55, 87,16,34, 45, 59, 20, 59.

Q20

The management of ABC company is considering the question of marketing a new product. The fixed cost
required in the project is Rs 4,000
3 factors are uncertain viz, the selling price, variable cost and the annual sales volume. The product has a life
of only one year. The management has the data on these 3 factors as under:
Selling price Rs
3
4
5

Probability
0.2
0.5
0.3

Variable cost Rs
1
2
3

Probability
0.3
0.6
0.1

Sales volume (units)


2,000
3,000
5,000

Probability
0.3
0.3
0.4

Considering the following sequence of thirty random numbers:


81, 32, 60. 04, 46, 31, 67, 25, 24, 10, 40, 02, 39, 68, 08, 59, 66, 90, 12, 64, 79, 31, 86, 68, 82, 89, 25, 11, 98,
16
Using the sequence (first 3 random numbers for the first trial etc) simulate the average profit for the above
project on the basis of 10 trials
n94
Q21

The Ever alert-Ltd.: which has a satisfactory preventive maintenance system in its plant, has installed a new
Hot Air Generator based on electricity instead of fuel oil for drying its finished products. The Hot Air Generator
requires periodicity shutdown maintenance. If.the shut down is scheduled yearly, the cost of maintenance will
be as under:
Maintenance cost (Rs)
Probability
15,000
0.3
20,000
0.4
25,000
0.3
The costs are expected to be almost linear i.e. if the shutdown is scheduled twice a year the maintenance cost
will be double.
There is no previous experience regarding the time taken between break downs. Costs associated with break
down will vary depending upon the periodicity of maintenance. The probability distribution of break down cost
is estimated as under:
Breakdown costs Rs per annum
Shut down once a year
Shut down twice a year
75,000
0.2
0.5
80,000
0.5
0.3
1,00,000
0.3
0.2
Simulate the total costs maintenance and breakdown cost and recommend whether shutdown overhauling
should be restored to once a year or twice a year?

65

PRAVINN MAHAJAN CLASSES 9871255244, 8800684854

Q22

A doctor who has introduced an appointments system for daily consultations has derived the following
information regarding patient punctually
Minutes early
3
6%
2
29%
1
41%
On time
Minutes late

12%
1

7%
5%
The doctor times his consultations over a period, and derives the following frequency distribution:
Minutes
12
10%
13
15%
14
28%
15
34%
16
13%
The doctor would like to issue appointments at 15 minutes interval and wishes to have an idea of his idle time,
the patient a waiting time, and whether he can complete his appointments on schedule.
Simulate sixteen consultations and derive the required information
Given the following series of random numbers:
17
14
50
40
83
13
94
08
49
98
79
51
43
74
92
24
09
21
40
12
46
91
09
05
95
44
52
79
91
53
15
16

66

PRAVINN MAHAJAN CLASSES 9871255244, 8800684854

PERT/ CPM
Q

Enumerate basic Assumptions of PERT/ CPM


1. A project can be sub-divided into a set of predictable independent activities
2. The precedence relationships of project activities can be completely represented by a noncyclical network graph in which each activity connects directly into its immediate successors.
3. activity times may be estimated either as single-point estimates or as three-point PERT estimates
and are independent of each other
4. In PERT/CPM model, activity duration is assumed to follow the beta distribution. The
standard deviation of the distribution is assumed to be 1/6th of its range; the mean is
approximated by 1/6( to + 4tm + tp) and the variances in length of a project is assumed to be
equal to the sum of the variances of activities on the critical path.

5. The CPM Model also has its special assumptions. One is; the duration of an activity is
linearly (and inversely) related to the cost of resources applied to the activity.

Distinguish between PERT and CPM


The PERT and CPM models are similar in terms of their basic structure, rationale and mode of analysis.
However, there are certain distinctions between PERT and CPM networks which are enumerated below:
1.

CPM is activity oriented i.e. CPM network is built on the basis of activities. Also results
of various calculations are considered in terms of activities of the project. On the other hand,
PERT is event oriented.

2.

CPM is a deterministic model i.e. it does not take into account the uncertainties involved in the
estimation of time for execution of a job or an activity. It completely ignores the probabilistic
element of the problem. PERT, however, is a probabilistic model. It uses three estimates of the
activity time; optimistic, pessimistic and most likely; with a view to take into account time
uncertainty. Thus, the expected duration for each activity is probabilistic and expected duration
indicates that there is fifty per probability of getting the job done within that time.

3.

CPM places dual emphasis on time and cost and evaluates the trade-off between project cost
and project time. By deploying additional resources, it allows the critical path project manager to
manipulate project duration within certain limits so that project duration can be shortened at an
optimal cost. On the other hand, PERT is primarily concerned with time. It helps the manager
to schedule and coordinate various activities so that the project can be completed on scheduled
time.

4.

CPM is commonly used for those projects which are repetitive in nature and where one
has prior experience of handling similar projects. PERT is generally used for those projects
where time required to complete various activities are not known as prior. Thus PERT is
widely used for Planning and scheduling research and development projects.
.

67

PRAVINN MAHAJAN CLASSES 9871255244, 8800684854

Basic steps involved in drawing a CPM/PERT network are :


(i)
(ii)
(iii)
(iv)
(v)
Q

Breaking up of the entire project into smaller systems known as tasks.


For each task ascertain the activities and events to be performed.
For each activity determine the preceding and succeeding activities.
For each activity determine or estimate the time and other resources needed.
Draw a network depicting the assembly of tasks into a project.

Explain the following in the context of a network:


a.
b.

N10

M08, M09

Critical path
Dummy activity

A Dummy Activity is a hypothetical activity which consumes no resources and time. Such an activity is
represented by dotted lines and is inserted in the network to clarify activity pattern under the following
situations:
(i)
It is created to make activities with common starting and finishing events distinguishable easily.
(II)
To Identify and maintain the proper precedence relationship between activities that are not
connected by any events.
(III)
To bring all "loose ends' to a single initial and a single terminal event In each network using
dummies, If required.
Critical Path: The longest time duration path is known as critical path & represented by thick line or
double lines. All activities lying in this critical path are called critical activities. Any delay in their execution
will lead to a delay in the completion of the entire project. .
Sometimes there may be more than one critical paths as they have same highest time duration. In that
case, the critical path having highest variance is called principle critical path.
Q

Define proiect. State some of its characteristics.

N91

A project can be defined as a set of activities or jobs that are performed in a certain sequence determined
logically or technologically and it has to be completed within (I) a specified time, (ii) a specified cost and
(iii) meeting the performance standards. A project is a new work for which organisation has no preliminary
experience.
Examples of a project from fairly diverse fields could be cited. Some of them are given below:
1. Introducing a new product in the market. .
2. Construction of a new bridge over a river or construction of a 25 -- storied building.
3. Executing a large and complex order on jobbing production. 4. Sending a space craft to the mars.
All these projects are characterized by the following set of common implications, although they pertain to
widely different fields.
(i) The large-scale characteristic : These projects are generally unusually large and complex. Thousands
of suppliers, workers and other categories of persons are involved and their efforts have to be .coordinate for completion of the project.
(ii) The non-recurring characteristic: These projects are generally of a one time nature. Neither in the past,
nor in the future they are likely to be undertaken substantially in the same form.
(iii) Uncertain and critical dates : Duration of the various activities involved in such projects are usually
uncertain. Further in such type of projects, many critical dates exist by which operations must be
completed in order to complete the entire project on schedule.
(iv) Completion dead line : The fourth distinct feature of these projects is that there is dead line for the
completion of the entire project. In case of any delay in the completion of the project, some penalty is
levied for such delay beyond the dead line.

68

PRAVINN MAHAJAN CLASSES 9871255244, 8800684854

Define various types of floats.


The four types of floats used in network analysis are briefly explained below:
(i) Total float:

The total float of an activity represents the amount of time by which an activity
can be delayed, without delaying the project completion date. In other words, it
refers to the amount of free time associated with an activity which can be used
before, during or after the performance of this activity. Total float is the positive
difference between the earliest finish time and the latest finish time, or the
positive difference between the earliest start time and the latest start time of an
activity depending upon which way it is defined.

(ii) Interfering float:

Utilization of the float of an activity may affect the float times of the other activities
in the network. Interfering float is that part of the total float which causes a
reduction in the float of the successor activities. It is the difference between the
latest finish time of the activity in question and the earliest starting time of the
following activity or zero, whichever is larger. It indicates the portion of the float of
an activity which cannot be consumed without affecting adversely the float of the
subsequent activities.

(iii) Free Float:

Free float is that portion of the total float within which an activity can be
manipulated without affecting the float of subsequent activities. It is computed for
an activity by subtracting the head event slack from its total float. The head event
slack is the difference between its latest and earliest event timings that is its (LE).

(iv) Independent Float: It is that portion of the total float within which an activity can be delayed for start
without affecting floats of the preceding activities. It is computed by subtracting
the tail event slack from the free float. If the result is negative, it is taken as zero.
Last column of table above gives independent floats of various activities of the
network
Q

Explain the terms Resource Smoothing and Resource Leveling.

M02, N02

Resource smoothing :

M99

It is a technique used for smoothening peak resource requirements during different periods of a project
net work. The total product duration is maintained at the minimum level. The constraint is on the project
duration time. It helps to estimate the total resource requirements for various projects. In resource
smoothing, time scaled diagram of various activities of a project and their floats along with their resource
requirements are used. The period of maximum demand for resources are identified and non critical
activities during these periods are staggered by rescheduling them according to their floats for balancing
the resource requirements.
Resource Leveling

M00

It is a net work technique used for Reducing the requirement of a particular resource due to its scarcity. It
utilizes the large floats available on non critical activities and cuts down the demand on resources. The
maximum demand of a resource should not exceed the available limit at any point of time. Non critical
activities are rescheduled by utilizing their floats.

69

PRAVINN MAHAJAN CLASSES 9871255244, 8800684854

Q1

Draw a network for the following


a.

b.
Activity

Preceding activity

A
B
C
D

c.

Activity

A,B
A

g.

i.

Activity
A
B
C
D
E
F

Preceding activity
A,B
B,C
B

Activity

Preceding activity

Activities

d.

B
C
B,C

A
B
C
D
E
F

f.

1 3,

1 4,

2 5,

70

B
A,B

Activity
A
B
C
D
E
F
G
H

Preceding activity
C
A,B
B
B,C
E,F,G

Activity
A
B
C
D
E
F

Preceding activity
A
A,B
A,B,C

Activity
A
B
C
D
E
F
G

h.

A
A,B
B,C

1 2,

Preceding Activity

A
B
C
D

Preceding activity

A
B
C
D
E
F

e.

Activity

3 6,

3 7,

Preceding activity
B
B
B
E
A,D,C
4 7,

5 8,

6 8,

7 9,

8 9.

PRAVINN MAHAJAN CLASSES 9871255244, 8800684854

J.
Activity
A
B
C
D
E
F
J

Preceding Activity
---A,B
B,C
A,B
C

Activity
H
I
J
K
L
M
N

Preceding Activity
D,E,F
D
G
G
H,J
K
I,L

Q2

A project has 14 activities A through M. The relationships which obtain among these activities are given here:
construct the network and number them.
A is the first operation
B and C can be performed in parallel and are immediate successors to A
D,E and F follow B
G follows E
H follows D, but it cannot be started until E is complete
I and J succeed G
F and J precede K
H and I precede L
M succeeds L and K
The Last operation N succeeds M and C.

Q3

Construct the network diagram comprising activities B, C,..Q and N Such that the following constraints are
satisfied:
B < E,F; C < G,L; E,G < H; L,H < I; L < M; H < N; H < J; I,J < P; P < Q.
The notation X < Y means that the activity X must be finished before Y can begin.

Q4

A project consists of a series of tasks labeled A, B,..H, I with the following relationship ( W < X,Y means
X,Y can start until W is completed; X, Y < W means W cannot start until both X and Y are completed).
With this notation construct the network diagram having the following constraints:
A<D,E;
B,D<F;
C<G;
B<H;
F,G<I.

Q5

Draw the network for the following


Activities
A
B
C
Predecessors -

Q6

D
A

E
B

F
C

G
B,D

H
C,E

Construct the network from the following information:


(a)
A,B and C, the first activities of the project can be executed concurrently.
(b)
D,E and F follow A
(c)
I and G follow both B and D
(d)
H follows both C and G
(e)
K and L follow I
(f)
J succeeds both E and H
(g)
M and N succeeds F, but cannot start until both E and H are completed
(h)
O succeeds both M and I
(i)
P succeeds J, L and O
(j)
K, N and P are terminal activities of the project.

71

I
F

J
F

K
E,G,I

L
H,J

PRAVINN MAHAJAN CLASSES 9871255244, 8800684854

Q7

Q8

(a)

The following list of activities must be accomplished in order to complete construction project
_____________________________________________________
Activities
Time (in weeks)
Predecessors
A
3
none
B
8
none
C
4
A,B
D
2
B
E
1
A
F
7
C
G
5
E,F
H
6
D,F
I
8
G,H
J
9
I
Construct a network diagram for this project. Find critical path and duration of the project.

(b)

Assume that you are the project manager of the project mentioned above. The project has progressed
to the end of the week 10 and the status is as follows:
Activities completed
A,B,E
Activities not started
C,D,F,G,H,I,J
If no managerial action is taken at all, when will the project get completed?
What action might you take to get the project back to a schedule that can be completed by the end
of the week 42?
M93

Name the events and determine the earliest time and latest time for each event and determine the critical path

I
4

C
3
A

D4

4
B
5

J 4
F
6

5
L
3

H 4

E
3
G
5

72

PRAVINN MAHAJAN CLASSES 9871255244, 8800684854

Q9

Find the critical path and the project completion time. Prepare an activity schedule showing the ES, EF, LS, LF
and float for each activity.

D
3
A
2

6
K
6

G
4

B
7
C
8

10

J
5

H
6
Q10

Given is the following information regarding a project:


Activity
Dependence
Duration (Days)

A
3

B
4

C
2

D
AB
5

E
B
1

F
B
3

G
FC
6

H
B
4

I
EH
4

Draw the Network Diagram and identify the Critical Path and Project Duration.
Find the 3 types of Float (viz. Total, Free and Independent) for each activity.
Q11

J
EH
2

K
CDFJ
1

L
K
5

N94

Consider the following project.


Activity
1-2
1-3
1-4
2-3
2-4
2-5

Duration
4
7
10
3
8
11

Activity
2-6
3-5
3-6
4-5
5-6
5-7
6-7

Duration
18
10
16
9
6
11
8

Calculate Early start, Early Finish, Latest start, Latest Finish, Total Float, Free Float for each activity
Identify the critical path and project duration.
If the project manager finds that either of the activities 2-6 or 4-5 can each be speeded up by 2
days at the same cost, which of the 2 activities should be speeded up? Explain.
Assuming the time estimates in Days given represent the expected duration based on three time
estimates and suppose the variance along the critical path is 81 days, What is the probability that
the project will be completed within 33 days and within 44 days?
N90

73

PRAVINN MAHAJAN CLASSES 9871255244, 8800684854

Q12

Q13

Q14

A project schedule has the following characteristics:


Activity
Time
Activity
1-2
4
5-6
1-3
1
5-7
2-4
1
6-8
3-4
1
7-8
3-5
6
8-9
4-9
5
8-10
9-10
Construct the PERT network
Compute E and L for each event
Float for each activity
Find critical path and its duration

M00

Given the following information:


Activity
A
B C
D E
F
G
Immediate predecessors - A
B
C,F
Duration
9
4 15
10 7
5
4
Determine ES, EF, LS, LF, IF and FF for each of the activities.

H
D,G
4

I
E,G
3

J
H,I
3

K
D
10

L
J,K
6

For a small project of 12 activities, the details are given below. Draw the network and compute earliest
occurrence time, latest occurrence time, critical activities and project completion time:
nd5
Activity
A
Dependence
Duration Days 9

Q15

Time
4
8
1
2
1
8
7

B
4

C
7

D
B,C
8

E
A
7

F
C
5

G
E
10

H
E
8

I
D,F,H
6

J
E
9

K
I,J
10

L
G
2

Bring out the dependency relationships from the following network .

8
Q

K
L

10

M
4

74

M
L
1

PRAVINN MAHAJAN CLASSES 9871255244, 8800684854

Q16

The number of days of total float (TF), earliest start times (EST) and duration in days are given for some
of the following activities.
Activity
12
13
14
24
25
36
46
57
67
68
78
89
Draw the network

TF

EST

0
0
5
0
1
2
0
3

Duration

4
5
12
12
23

2
0

23
30

6
N11

List the paths with their corresponding durations and state when the project can be completed.
Q17

Points out the errors in the networks given below, going by the usual conventions while drawing a networks to use
CPM
M11
a

7
1

6
5
8

75

PRAVINN MAHAJAN CLASSES 9871255244, 8800684854

b.

A
F

15
J

H 6 G
C

B
1

3
E

8
L

12

14

13

10

11

c.

K
2
3
L
1

R
N

U
8

O
5

Q18

The following table shows for each activity- the normal time, the shortest time in which the activity can be
completed of a building contract and the cost per day for reducing the time of each activity. The contract
includes a penalty clause of Rs 100 per day over 17 days. The overhead cost per day is Rs 160.
Activity
1-2
1-3
1-4
2-4
2-5
3-6
4-6
5-6

Normal time in days


6
8
5
3
5
12
8
6

Shortest time in days


4
4
3
3
3
8
5
5

76

Cost reduction per day


80
90
30
40
200
50
-

PRAVINN MAHAJAN CLASSES 9871255244, 8800684854

The cost of completing the eight activities in the normal time is Rs 6,500. Calculate
a) The normal duration of the project, its cost and its critical path.
b) The lowest cost and associated time.
c) The shortest time and associated cost.
Q19

A small project consists of six activities with following information:


Normal duration(
Crash duration (days) Crashing cost (Rs per day
days)
9
6
20
8
5
25
15
10
30
5
3
10
10
6
15
2
1
40
Draw the network and obtain normal and minimum project durations
Determine the minimum crashing cost of schedules ranging from normal length down to
and including the minimum length schedule. That is if L = Length of schedule, find the
cost of schedules which are L, L-1, L-2 and so on.
Overhead costs total Rs 60 per day. What is the optimum length schedule in terms of both
crashing and overhead cost? List the schedule duration of each job for your solution. M02

Activity(i-j)
1-2
1-3
1-4
2-4
3-4
4-5
a)
b)

c)

Q20

The following is a table showing details of a project (time in weeks, cost in thousands)
Task

Immediate predecessor

Normal time (weeks)

Crash cost (Rs 000)

Time(weeks)

Cost (Rs 000)

10

20

30

B
8
15
6
20
C
B
5
8
4
14
D
B
6
11
4
15
E
B
8
9
5
15
F
E
5
5
4
8
G
A,D,C
12
3
8
4
Indirect cost is Rs 400 per day. Find the optimum duration and the associated minimum project cost.
Q21

The normal time, crash time and crashing cost per day are given for the following network:

77

PRAVINN MAHAJAN CLASSES 9871255244, 8800684854

i.

Activity
Normal time (days)
Crash time (days)
Crashing cost (Rs per day)
1-2
18
14
40
1-3
23
22
20
2-3
8
5
60
2-4
10
6
40
3-4
3
2
80
4-5
8
6
50
Crash the project duration in steps and arrive at the minimum duration. What will be the critical path

and the cost of crashing?


ii.

Q22

If there is an indirect cost of Rs 70 per day, what will be the optimal project duration and the cost of
crashing?
N08

A project is composed of seven activities as per the details given below:


Activity
1-2
1-3
1-4
2-3
2-5
3-5
4-5

Normal time (days)


4
2
5
7
7
2
5

Crash time (days)


3
2
4
5
6
1
4

Normal cost(Rs)
1500
1000
1875
1000
2000
1250
1500

Crash cost (Rs)


2,000
1,000
2250
1500
2500
1625
2125

Indirect. cost per days of the project is Rs 500


Required

Draw the project network


Determine the critical path and its duration
Find the optimum duration and the resultant cost of the project

5 weeks

8 5 weeks 9

3 weeks

w
ee

ks

A network is given below:


(i)
Name the paths and give their total duration
(ii)
Give three different ways of reducing the project above duratiol1 by 4 days.

s
3

2 weeks

3 weeks

78

s
ek
we

ek
we

we
e

ks

2 w eeks

Q23

(a)
(b)
(c)

M04

N06

PRAVINN MAHAJAN CLASSES 9871255244, 8800684854

Q24

A project has 7 activities. The relevant data about these activities is given below:
Activity
A
B
C
D
E
F
G
1.
2.

Q25

Dependence
A
A
A
B,C
C,D
E,F

Normal duration
7
4
5
6
7
5
6

Crash duration
5
2
5
4
4
2
4

Normal cost
500
400
500
800
700
800
800

Find out normal duration and the minimum duration?


What is the percentage increase in cost to complete the project in 21 days?

N97

The following table gives data on Normal time and cost and Crash time and cost of the project
a.
Draw the network and identify the critical path.
b.
What is the normal project duration and associated cost?
c.
Find out total float for each activity.
d.
Crash the relevant activities systematically and determine the optimum project time and cost.
Activity

Normal

Crash

Time (week)
1-2
2-3
2-4
2-5
3-5
4-5
5-6
6-7
6-8
7-8

Cost (Rs)

3
3
7
9
5
0
6
4
13
10

Time (week)

300
30
420
720
250
0
320
400
780
1000
4220

Cost (Rs)

2
3
5
7
4
0
4
3
10
9

400
30
580
810
300
0
410
470
900
900
4800

Indirect cost are Rs 50 per week


Q26

Crash cost
900
600
500
1000
1000
1400
1600

N09

The following network gives the duration in days for each activity:

2
1

7
3

3
1

4
4

79

2
6

PRAVINN MAHAJAN CLASSES 9871255244, 8800684854

(i)
(ii)

Q27

You are required to list the critical paths.


Given that each activity can be crashed by a maximum of one day, choose to crash any four
activities so that the project duration is reduced by 2 days.
N09

A small project is composed of seven activities, whose time estimates (in days) are listed below. Activities
are identified by their beginning (i) and ending (j) node numbers.
Activity(i-j) 1-2
1-3
1-4
2-5
3-5
4-6
5-6
Duration to 2
2
4
2
4
4
6
Duration tm 2
8
4
2
10
10
12
Duration to 14
14
116
2
28
16
30
1. Draw the project network
2. Find the expected duration and variance for each activity. What is the expected project length?
3.
If the project due date is 38 days, what is the probability of meeting the due date.
N05

Q28

If the critical path of a project is 20 months along with a standard deviation of 4 months, what is the
probability that the project will be completed within: (a) 20 months (b) 18 months (c) 24 months?

Q29

A civil engineering firm has to bid for the construction of a building. The activities and the time estimates
in days are given below:
Activity
To
Tp
Tm
1-2
15
17
25
2-3
14
18
22
2-4
13
15
17
2-8
16'
19
28
3-4 (dummy)
3-5
15
18
27
4-6
13
17
21
5-7(dummy)
5-9
14
18
22
6-7(dummy)
6-8(dummy)
7-9
16
20
42
8-9
14
16
24
The policy of the firm with respect to submitting bids is to bid the minimum amount that will provide a 95%
probability of at best breakeven. The fixed costs for the project are Rs 8 lakhs and the variable costs are Rs
9,000 every day spent working on the project. The duration given above is in days. What amount should the
firm bid under this policy (Take upto 2 decimal places for calculations)
M90

Q30

A mother notes that her teenage son uses the telephone, he takes no less than 10 minutes for a call and
sometimes as much as one hour. Twenty minutes calls are more frequent than calls of any other duration. If
the son's phone calls were an activity in PERT project
(a) What would be phone calls expected duration.
(b) What would be its variance?
(c) In scheduling the project, how much time would be allocated for the phone call?

Q31

One of the activities in the PERT project has an unexpected duration of 12 weeks with a standard deviation of
2 weeks. The most likely time estimate of this activity is 12 weeks. Calculate the optimistic and Pessimistic
time estimates for this activity.

80

PRAVINN MAHAJAN CLASSES 9871255244, 8800684854

Q32

A project consists of following activities whose time estimates are given below
Activity
Optimistic
Most likely
Pessimistic
a)
(b)
(c)
(d)
(e)

Q33

1-2
3
6
15

1-3
2
5
14

2-5
2
5
8

2-6
5
11
17

3-6
3
6
15

4-7
3
9
27

5-7
1
4
7

Draw a project network.


Find the expected duration and variance of each activity.
Determine the critical path and the expected project duration.
What is the probability that the project will be completed in 38 weeks?
What project duration will have 95% chance of completion (Z 0.95 = 1.65)
Given Z
0.21
0.41
0.82

0.0832
0.1591
0.2939

6-7
4
19
28

M03

A project consists of activities A to I. The notation X <Y means that the activity X must be completed before Y
can start and X, Y < W means that W will start only after the completion of X and Y. With this notation,
construct a network diagram for the following constraints:
A<D, A<E, B<F, C<G, D<H
and
E, F < I
The project has the following time schedules (weeks) for the above activities:
Activity
Least time (to)
Greatest time (tp)
Most likely (tm)
(a)
(b)
(c)

Q34

1-4
6
12
30

A
5
10
8

B
18
22
20

C
26
40
33

D
16
20
18

E
15
25
20

F
6
12
9

G
7
12
10

H
7
9
8

I
3
5
4

Determine the expected task times and their variance


Calculate the earliest and latest expected times to reach each node
Find the critical path and the probability of a node occurring at the proposed completion date if
the original contract time of completing the project is 41.5 weeks.

The time estimate (in weeks) for the activities of a PERT network are given below:
Activity
To
Tm
Tp
1-2
1
1
7
1-3
1
4
7
1-4
2
2
8
2-5
1
1
1
3-5
2
5
14
4-6
2
5
8
5-6
3
6
15
(a)
Draw the project network and identify all the paths through it.
(b)
Determine the expected project length.
(c)
Calculate the standard deviation and variance of the project length.
(d)
What is the probability that the project will be completed?
1.
at least 4 weeks earlier than expected time? .
2.
no more that 4 weeks later than expected time?
(e)
If the project due date is 19 weeks, what is the probability of not meeting the due date?
(f)
The probability that the project will be completed on schedule if the scheduled completion time is 20
weeks. .
(g)
What should be the scheduled completion time for the probability of completion to be 90%?
N91

81

PRAVINN MAHAJAN CLASSES 9871255244, 8800684854

Q35

An Engineering Project has the following activities; whose time estimates are listed below:

(a)
(b)
(c)
(d)

Estimated Duration (in months)


Optimistic
Most Likely
Pessimistic

1-2
1-3
1-4
2-5
3-5
4-6
5-6

2
2
4
2
4
4
6

2
8
4
2
10
10
12

14
14
16
2
28
16
30

Draw the project network and find the critical path.


Find the expected duration and variance for each activity. What is the expected project length?
Calculate the variance and standard deviation of the project length.
What is the probability that the project will be completed at least eight months earlier than
expected time?
If the project due date is 38 months, what is the probability of not meeting the due date?
Given:
z
0.50
0.67
1.00
1.33
2.00
P
0.3085
0.2514
0.1587
0.0918
0.0228
M03

(e)

Q36

Activity (i-j)

A company is launching a new product and has made estimates of the time for the various activities
associated with the launch as follows:
Activity
A
B
C
D
E
F
G
H
I

Predecessor
NONE
NONE
A, B
B
A
C
E, F
D, F
G, H

Times (Davs)
optimistic
Most likely
1
3
3
4
1
3
3
3
1
2
2
5
2
3
2
2
10
10

Pessimistic
5
5
11
9
3
14
4
2
10

Required:

i.
ii.
iii.
iv.
v.
Q37.

Draw the network diagram


Calculate the expected time and variance of each activity.
Find out the expected length of critical path and 'its standard deviation.
Find the probability that the launching will be completed in 27days.
Find the duration, which has 95% probability of completion.

N09

Consider the schedule of activities and related information as given below, for the construction of a plant:
Activity
(Months)
1-2
2-3
3-6
2-4
1-5
5-6
4-6
5-7
7-8
6-8

Expected Time
(Millions of Rs.)
4
2
3
6
2
5
9
7
10
1

82

Variance
1
1
1
2
1
1
5
8
16
1

Expected Cost
5
3
4
9
2
12
20
7
14
4

PRAVINN MAHAJAN CLASSES 9871255244, 8800684854

Assuming that the cost and time required for one activity is independent of the time an cost of any other
activity are expected to follow normal distribution.
Draw a network based on the above data and calculate:
(a)
Critical path
(b)
Expected cost of construction of the plant.
(c)
Expected time required to build the plant.
M01
(d)
The standard deviation of the expected time
Q38

The following information is given:


Activity

(1-2)

(2-3)

(2-4)

(3-5)

(4-6)

(5-6)

(5-7)

(6-7)

Pessimistic time (in weeks)

Most likely time (in weeks)

Optimistic time (in week's)

Draw the Network diagram for the above. Calculate:


1)
Variance of each activity.
2)
Critical path and expected project length.
3)
The probability that the project will be completed in 23 weeks.
Given that:
Z value
1.90
1.91
1.92
1.93
1.94
Probability
0.9713 0.9719 0.9726 .9732
0.9738
Q39

A project consists of seven activities and the time estimates of the activities are furnished as under:
Activity

Optimistic
Days
4
3
4
5
8
4
2

1-2
1-3
1-4
2-5
3-.5
4-6
5-6
Required

(i)
(ii)
(iii)

Pessimistic
Days
16
9
16
5
32
16
8

Draw the network diagram


Identify the critical path n its duration
What is the probability that project will be completed in 5 days earlier than the
critical path duration?
What project duration will provide 95% confidence level of completion
(Z 0.95 = 1.65)
N98

(iv)
Q40

Most likely
Days
10
6
7
5
11
10
5

Consider the following project


Activity
Predecessor

A
None

Estimated time in weeks


Optimistic
3
Most Likely
6
Pessimistic
9
a)
(b)

B
None

2
5
8

C
A

2
4
6

D
B

E
B

2
3
10

1
3
11

F
C,D

G
E

4
6
8

1
5
15

Draw the Network Diagram, find the critical path and its duration.
Tabulate all kinds of floats and also compute the variance for all activities.

83

PRAVINN MAHAJAN CLASSES 9871255244, 8800684854

(c)
(d)
(e)
(f)
(g)

What is the probability that the project will be completed by 18 weeks?


What is the probability that the project will be completed by 12 weeks?
Estimate the chance that the probability will take more than 19 weeks.
Estimate the chance that the probability will take more than 14 weeks.
If the Project Manager wants to be 95% sure of completion, how many weeks before the due
date should he commence the project?
The Company's Competitor wants to give the Company only a 10% chance of completing the
project. What should be the commencement date for such a chance?

(h)

Q41

Given the following project network, determine


Earliest expected completion time for each event
Latest allowable completion time for each event
Slack time for each event
Critical path
Probability that the project will be completed on schedule, if the scheduled completion time is 38
Activity
Duration (weeks)
Predecessors
Crew Size (workers)

A
4
None
4

2
ET=4
= 2

B
7
None
2

C
3
A
2

D
3
A
4

ET=20
= 10

F
2
B
3

G
2
D,E
3

H
3
F,G
4

5
ET=8

ET=10

ET=20
= 1

E
2
B
6

=1

= 4
4

ET=5
ET=3 = 2

= 3
3
Q42

ET=8
= 4

ET=6
= 2

ET=10
= 8
6

The Chennai Construction company is bidding on a contract to install a line of microwave towers. It has
identified the following activities, along with their expected times, predecessor restrictions and worker
requirements. The contract specifies that the project should be completed in 14 weeks. The company will
assign a fixed number of workers to the project for the entire duration, and so it would like to ensure that the
minimum number of workers is assigned and that the project will be completed in 14 weeks. Find a schedule
which will do this.
M95

84

PRAVINN MAHAJAN CLASSES 9871255244, 8800684854

Q43

Following are the manpower requirements for each activity in a project:


Activity
Normal Time (day)
Man power required per day
1-2
10
2
1-3
11
3
2-4
13
4
2-6
14
3
3-4
10
1
4~
7
3
4-6
17
3
5-7
13
5
6~
9
8
7-8
1
11
(a)
Draw a network and find out total float and free float for each activity.
(b)
The contractor stipulates that during the first 26 days only 4 to 5 men and during remaining days
8 to 11 men only can be made available. Rearrange the activities suitably for leveling the manpower
resources, satisfying the 'above condition

Q44

A project consists of 10 activities, each of which requires either or both of the 2 types of resources R 1 and
R2 for its performance. The duration of the activities and their resource requirements are as follows :
Activity

Duration (days)

Resource requirement
R1
R2
3
2
6
4
4
2
2
4
4
3
2
1
3
4
5

1-2
3
1-3
2
1-4
6
2-6
4
3-5
2
4-5
1
4-8
4
5-7
3
6-7
2
7-8
4
Resource availability:
R1 = 7units, R2 = 5 units
Determine the duration of the project under the given resource constraint. If the resources were not a
problem, how long would the project take to complete in the normal course?

Q45

For a project consisting of several activities, the durations and required resources for carrying out each of
the activities and their availabilities are given below:
(a) Draw the network, identify critical path and compute the total float for each of the .activities.
(b) Find the project completion time under the given resource constraints .

Activity
1-2
1-3
1-4
2-4
2-5
3-4
3-5
4-5

Resources required
Equipment
Operators
X
30
Y
20
Z
20
X
30
Z
20
Y
20
Y
20
X
30

85

Duration (Days)
4
3
6
4
8
4
4
6

PRAVINN MAHAJAN CLASSES 9871255244, 8800684854

Q46

The following information is available:


Activity
A
B
C
D
E
F
G
(a)
(b)
(c)

Q47

No of men required per day


2
3
5
3
2
3
8

Draw the network and find ,the critical path.


What is the peak requirement of Manpower? On which day(s) will this occur?
If the maximum labour available on any day is only 10, when can the project be completed?

M08

A network with the following activity durations and manpower requirement is given. Analyze the project from
point of view of resources to bring out the necessary steps involved in the analysis and smoothing of
resources.
Activity:
Duration (weeks):
No. of Men required

Q48

No. of days
4
2
8
6
4
1
1

1-2
1-3
1-4
2-6
3-5
5-6
4-6

1-2
2
4

2-3
3
3

2-4
4
3

3-5
2
5

4-6
4
3

4-7
3
4

5-8
6
3

6-8
6
6

7-9
5
2

8-10
4
2

9-10
4
9

1-4
6

2-5
16

4-7
19

3-6
24

5-7
9

6-8
7

7-8
8

A company had planned its operations as follows:


Activity:
Duration (Days)
(i)
(ii)

1-2
7

2-4
8

1-3
8

3-4
6

Draw the network and find the critical paths'.


After 15 days of working, the following progress is noted:
(a)
(b)
(c)
(d)

Activities 1-2,1-3 and 14 completed as per original schedule.


Activity 2-4 is in progress and will be completed in 4 more days.
Activity 3-6 is in progress and will need 17 more days to complete.
The staff at activity 3-6 is specialized. They are directed to complete 3-6 an undertake an
Activity 6-7, which will require 7days. This rearrangement arose due to a modification in a
specialization.
(e)
Activity 6-8 will be completed in 4 days instead of the originally planned 7 days.
(f)
There is no change in the other activities.
Update the network diagram after 15 days of start of work based on the assumption given above.
Indicate the revised critical paths along with their duration.
M07

86

PRAVINN MAHAJAN CLASSES 9871255244, 8800684854

87

Das könnte Ihnen auch gefallen